You are on page 1of 69

SOLUTION MANUAL FOR DYNAMIC BUSINESS LAW

THE ESSENTIALS 3RD EDITION KUBASEK BROWNE


HERRON DHOOGE BARKACS ISBN 007802384X
9780078023842
Full download link at:
Solution manual: https://testbankpack.com/p/solution-manual-for-dynamic-
business-law-the-essentials-3rd-edition-kubasek-browne-herron-dhooge-
barkacs-isbn-007802384x-9780078023842/
Test bank: https://testbankpack.com/p/test-bank-for-dynamic-business-law-
the-essentials-3rd-edition-kubasek-browne-herron-dhooge-barkacs-isbn-
007802384x-9780078023842/
Chapter 7: Tort Law

Chapter 08

Real, Personal, and Intellectual Property

True / False Questions

1. Real property includes land and everything permanently attached to it.

True False

2. The laws of intellectual property protect tangible property.

True False

3. Ownership of real property includes the airspace above the land but not the mineral rights below
the land.

True False

4. A landowner may be required to involuntarily transfer their property to the government.

True False

5. An easement is a revocable right to use some part of another's land for a specific purpose without
taking anything from it.

True False

8-1
Copyright © 2016 McGraw-Hill Education. All rights reserved. No reproduction or distribution without the prior written consent of
McGraw-Hill Education.
6. Subleasing of leased property by the tenant to another party is permissible unless specifically
prohibited by the lease.

True False

7. A license is a right to use another's property that is permanent and revocable.

True False

8. Condemnation is a legal process that is protected by the federal constitution and allows the
government to take private property only if the property is used for new schools or roads.

True False

9. All property that is not land or permanently affixed to land is personal property.

True False

10. A book is typically considered intellectual property.

True False

11. Personal property can be tangible property but not intangible property.

True False

12. A causa mortis gift may be revoked any time before the donor's death and is automatically
revoked if the donor recovers.

True False

13. To register a domain name on the Internet, an application must be filed with Network Solutions
Inc., which is funded by the National Science Foundation.

True False

14. A person who applies for a domain name on the Internet will be granted the domain name so long
as no one else holds that name.

True False

15. Trade dress refers to the overall appearance and image of a product and, is entitled to the same
protection as a trademark.

True False

16. Copyrights protect the ideas of creative ideas.

True False

17. Copyright is the protection of the expression of a fixed form that expresses ideas.

True False

8-2
Copyright © 2016 McGraw-Hill Education. All rights reserved. No reproduction or distribution without the prior written consent of
McGraw-Hill Education.
18. When copyright is at issue, proof that an item is similar is not necessarily infringement.

True False

19. A teacher cannot be held liable for copyright infringement under the Fair-Use Doctrine so long as
copies are only used for educational purposes.

True False

20. Tying arrangements are an illegal agreement in which the sale of one product is linked to the sale
of another.

True False

21. A patent grants the holder exclusive rights for 40 years.

True False

22. An invention may be considered a trade secret.

True False

23. Trade-secret protection is lost if another business discovers the trade secret in a lawful manner.

True False

24. In Germany land expropriated by the government reverts to its former owner if the government no
longer needs the property.

True False

25. In Kelo v. City of New London, the U.S. Supreme Court ruled that as a matter of law, the
government violates the Fifth Amendment by taking private property and selling it for private
development.

True False

26. Lost property and mislaid property are the same legal theories.

True False

Multiple Choice Questions

27. What are the three basic types of property?

A. Real, absolute, and personal


B. Real, intellectual, and land
C. Intellectual, land, and minerals
D. Real, personal, and intellectual
E. None of these choices.

8-3
Copyright © 2016 McGraw-Hill Education. All rights reserved. No reproduction or distribution without the prior written consent of
McGraw-Hill Education.
28. Land and everything permanently attached to it are known as:

A. Real property
B. Personal property
C. Absolute property
D. Listed property
E. Fee property

29. The right to enter onto property to remove underground materials is included in the ownership of
____________________.

A. subsurface rights
B. underground rights
C. earth rights
D. terrestrial rights
E. None of these, because there is no such right

30. The duration of one's ownership interest and the power one has over using the land depends on
the type of ______ one is said to hold.

A. condition
B. estate
C. tenancy
D. hold
E. acknowledgement

31. Real property ownership includes airspace above the land, water rights flowing across or beneath
the land, and ___________________.

A. trees growing next to the land


B. the right to divert the water to deprive landowners downstream from use of the water
C. walkways leading to the land
D. mineral rights under the land
E. roadways providing access to the land

32. A ______ estate is the most complete estate a person may have.

A. conditional estate
B. life estate
C. leasehold estate
D. future interest
E. fee simple absolute

8-4
Copyright © 2016 McGraw-Hill Education. All rights reserved. No reproduction or distribution without the prior written consent of
McGraw-Hill Education.
33. The owner of a ______ estate possesses the same interest as the owner of a fee simple
absolute, only this interest is subject to a condition that may result in termination of the interest.

A. conditional estate
B. life estate
C. leasehold estate
D. future interest
E. fee simple absolute

34. An ownership interest in which the holder has the right to possess the property until his or her
death is considered a ___________.

A. conditional estate
B. life estate
C. leasehold estate
D. future interest
E. fee simple absolute

35. A ______ is a person's present right to future property ownership and possession.

A. conditional estate
B. life estate
C. leasehold estate
D. future interest
E. fee simple absolute

36. An owner of a ______ has a possessory interest but not an ownership interest.

A. conditional estate
B. life estate
C. leasehold
D. future interest
E. fee simple absolute

37. Which of the following is false regarding property interests in Vietnam?

A. The constitution asserts that the state owns all the land.
B. If an individual wants to use land, he or she must pay tax on it as a form of rent.
C. Transference of property can occur only with the approval of a state official.
D. A new owner can never be given a longer term of right or more extensive rights over the land
than the original owner had.
E. The transferring owner determines the price for which property will be transferred.

8-5
Copyright © 2016 McGraw-Hill Education. All rights reserved. No reproduction or distribution without the prior written consent of
McGraw-Hill Education.
38. Which of the following are estates that do not include the right to possess the property?

A. Easements
B. Profits
C. Licenses
D. Easements, profits, and licenses
E. Easements and profits, but not licenses

39. A(n) ______ is an irrevocable right to use some part of another's land for a specific purpose
without taking anything from it.

A. appurtenance
B. profit
C. easement
D. fixation
E. joint use

40. A(n) ______ is the right to go onto someone's land and take part of the land or a product of it
away from the land.

A. appurtenance
B. profit
C. easement
D. fixation
E. joint use

41. Which of the following procedures must be followed in order to effectuate a voluntary transfer of
real property?

A. Execution
B. Delivery
C. Acceptance
D. Execution, delivery, acceptance, and recording
E. Execution and delivery, but not acceptance

42. Which of the following is the legal process by which a transfer of property is made by a
governmental entity against the protest of the property owner pursuant to the takings clause of
the Fifth Amendment?

A. Allocation
B. Appropriation
C. Condemnation
D. Substitution
E. Publication

8-6
Copyright © 2016 McGraw-Hill Education. All rights reserved. No reproduction or distribution without the prior written consent of
McGraw-Hill Education.
43. Which of the following consists of the fruits of one's mind?

A. Theoretical property
B. Cognitive property
C. Intellectual property
D. Protected property
E. Tradable property

44. An example of tangible property is

A. Furniture and cars


B. Furniture but not cars
C. Bank accounts
D. Bank accounts and stocks
E. Stocks and insurance policies

45. What type of gift is made by a person during his or her lifetime?

A. Adverse possession
B. Profit
C. An easement
D. Causa mortis
E. Inter vivos

46. What is the term for a gift that is made in contemplation of one's immediate death?

A. Adverse possession
B. Profit
C. An easement
D. Causa mortis
E. Inter vivos

47. A(n) ________ is a distinctive mark, word, design, picture, or arrangement that is used by a
producer in conjunction with a product and tends to cause consumers to identify the product with
the producer.

A. copyright
B. patent
C. trade secret
D. trademark
E. intellamark

8-7
Copyright © 2016 McGraw-Hill Education. All rights reserved. No reproduction or distribution without the prior written consent of
McGraw-Hill Education.
48. Which of the following is true regarding whether the shape of a product or package may be a
trademark?

A. The shape of a product may be a trademark if it is nonfunctional, but the shape of a package
may not be the subject of a trademark.
B. The shape of a product may be the subject of a trademark if it is functional, but the shape of
the package may not be the subject of a trademark.
C. The shape of a product or package may be a trademark if it is functional.
D. The shape of a product or the shape of a package may be a trademark if it is nonfunctional.
E. The shape of a package may be a trademark if it is nonfunctional, but the shape of a product
may not be the subject or a trademark.

49. Under which of the following is a trademark protected under state common law?

A. If it is used interstate
B. If it is used intrastate
C. If it is used commercially
D. If it is used intrastate and commercially
E. A trademark cannot be protected under state common law

50. To be protected in ____________ use, a trademark must be registered with the U.S. Patent
Office under the Lanham Act of 1947.

A. interstate
B. intrastate
C. interstate and intrastate
D. commercial
E. interstate, intrastate, and commercial

51. If a trademark is registered, what may the owner obtain in the event of infringement from a person
who used the trademark to pass off goods as being those of the mark owner?

A. Damages
B. An injunction prohibiting the infringer from using the mark
C. An additional amount of damages computed as a multiplier of 5 times the original damages
D. Damages, an injunction prohibiting the infringer from using the mark, and additional damages
based on a multiplier of 5 times the original damages
E. Damages and an injunction prohibiting the infringer from using the mark

8-8
Copyright © 2016 McGraw-Hill Education. All rights reserved. No reproduction or distribution without the prior written consent of
McGraw-Hill Education.
52. If a trademark is unregistered, which of the following may the holder recover when an infringer
uses the mark to pass off goods as being those of the mark owner?

A. Damages.
B. An injunction prohibiting the infringer from using the mark.
C. An additional amount of damages computed as a multiplier of 5 times the original damages.
D. Damages, an injunction prohibiting the infringer from using the mark, and additional damages
based on a multiplier of 5 times the original damages.
E. Damages and an injunction prohibiting the infringer from using the mark.

53. Once a trademark is registered, when must it initially be renewed?

A. Between the first and second years


B. Between the second and third years
C. Between the third and fourth years
D. Between the fourth and fifth years
E. Between the fifth and sixth years

54. After the initial renewal, assuming a trademark was initially registered after 1990, how often must
the trademark be renewed?

A. Every nine years


B. Every ten years
C. Every eleven years
D. Every twelve years
E. Every thirteen years

55. In which of the following orders of ascending strength may trademarks fall?

A. Generic, suggestive, descriptive, arbitrary, or fanciful


B. Descriptive, generic, suggestive, arbitrary, or fanciful
C. Arbitrary or fanciful, generic, descriptive, suggestive
D. Generic, descriptive, suggestive, arbitrary, or fanciful
E. Suggestive, generic, descriptive, arbitrary, or fanciful

56. Which of the following is the way people and businesses are located on the web?

A. Through product names


B. Through Internet names
C. Through domain names
D. Through trademark names
E. Through search engines

8-9
Copyright © 2016 McGraw-Hill Education. All rights reserved. No reproduction or distribution without the prior written consent of
McGraw-Hill Education.
57. With which of the following does a government website address end?

A. .gov
B. .edu
C. .net
D. .org
E. .com

58. Which of the following is responsible for registering domain names for Internet use?

A. Internet Solutions Inc.


B. Domain Names Co.
C. Internet Domain Inc.
D. Network Solutions Inc.
E. Findability Inc.

59. A registrant may lose registration of a domain name for Internet usage by not using it for more
than ___________ days.

A. 30
B. 60
C. 90
D. 120
E. 180

60. Which of the following provides that a portion of a copyrighted work may be reproduced for
purposes of criticism, comment, news reporting, teaching, scholarships, and research?

A. The limited-use doctrine


B. The copyright-use doctrine
C. The fair-use doctrine
D. The trade-use doctrine
E. None of these because there is no such provision

61. A patent protects a product, process, invention, machine, or a(n) ______.

A. trademark
B. copyright
C. plant produced by asexual reproduction
D. book that has not yet been submitted for copyright protection
E. idea

8-10
Copyright © 2016 McGraw-Hill Education. All rights reserved. No reproduction or distribution without the prior written consent of
McGraw-Hill Education.
62. Which of the following criteria must be satisfied in order for a patent, other than a design patent,
to be granted?

A. The object of the patent must be novel.


B. The object of the patent must be useful.
C. The object of the patent must be nonobvious.
D. The object of the patent must be novel but it does not have to be nonobvious.
E. The object of the patent must be novel, useful, and nonobvious.

63. When a patent is issued for an object, it gives its holder the exclusive right to produce, sell, and
use the object of the patent for __________ years from the date of application.

A. ten
B. twenty
C. thirty
D. forty
E. fifty

64. Which of the following is true regarding trade-secret protection?

A. A trade secret is protected from unlawful appropriation by competitors as long as it is kept


secret and consists of elements not generally known in the trade.
B. A trade secret is protected from unlawful appropriation by competitors for ten years so long as
no competitor gains access to the secret through lawful means.
C. A trade secret is protected from unlawful appropriation by competitors for twenty years so long
as no competitor gains access to the secret through lawful or unlawful means.
D. A trade secret is protected from unlawful appropriation by competitors for thirty years.
E. A trade secret is protected from unlawful appropriation by competitors for seventy years.

65. Which of the following is false regarding trade-secret protection?

A. Competitors may not legally discover trade secrets by doing reverse engineering.
B. Competitors may discover secrets by going on public tours of plants and observing the use of
the trade secret.
C. Lawful discovery of a trade secret means there is no longer a trade secret to be protected.
D. An invention may be considered a trade secret.
E. A design may be considered a trade secret.

66. To stop a competitor form using a trade secret, the plaintiff must prove that a trade secret actually
existed, that the defendant acquired the trade secret unlawfully and ___________.

A. it is the plaintiff's sole trade secret


B. the inventor successfully patented the trade secret
C. the defendant published the trade secret
D. the defendant used the trade secret without the plaintiff's permission
E. the plaintiff had the trade secret for less than 20 years

8-11
Copyright © 2016 McGraw-Hill Education. All rights reserved. No reproduction or distribution without the prior written consent of
McGraw-Hill Education.
67. In the intellectual property arena, the term ______ refers to the overall appearance and image of
a product.

A. product appearance
B. trade dress
C. design fabrication
D. trade appearance
E. product application

68. Which of the following is a type of property that can be touched?

A. Tangible
B. Intangible
C. Substantive
D. Productive
E. All of these

69. Bank accounts, stocks, and insurance policies are examples of which of the following types of
property?

A. Tangible
B. Intangible
C. Substantive
D. Productive
E. All of these

70. Which of the following is a way in which a gift differs from a purchase as a way of transferring
ownership?

A. Once delivered a gift may be taken back by the person who gave the gift, whereas that is not
true when an item is purchased.
B. No consideration is needed for a gift.
C. No written contract is needed for a gift, whereas it is needed for a purchase.
D. No certificate of title is needed for a gift, whereas it is needed for a purchase.
E. All of these.

71. Which of the following is needed for a valid gift?

A. Delivery
B. Donative intent
C. Acceptance
D. Delivery, donative intent, and acceptance
E. Delivery and donative intent, but not acceptance

8-12
Copyright © 2016 McGraw-Hill Education. All rights reserved. No reproduction or distribution without the prior written consent of
McGraw-Hill Education.
72. Property that the original owner has discarded is ______ property.

A. mislaid
B. tossed
C. discarded
D. abandoned
E. terminated

73. Which of the following is property that the true owner has unknowingly or accidentally dropped or
left somewhere?

A. Lost
B. Mislaid
C. Discarded
D. Abandoned
E. Terminated

74. Which of the following is property that the true owner has intentionally placed somewhere but has
forgotten its location?

A. Lost
B. Mislaid
C. Discarded
D. Abandoned
E. Terminated

75. Which of the following is true regarding lost property?

A. In most states the finder of lost property has title to the lost good against all including the
person who lost the property.
B. In most states the finder of lost property has title to the lost good against all except the person
who lost the property.
C. In most states the finder of lost property has no title to the lost good because the good is to be
turned over to the police department for later sale if the true owner cannot be located.
D. In all states the finder of property has an obligation to put a notice in the local paper looking for
the true owner before the finder of the property can claim ownership.
E. In most states the finder of lost property has title to the lost good against all except the person
who lost the property, but that right only arises after the finder of the property can establish that
the finder spent at least one-third of the fair market value of the property in locating the owner.

8-13
Copyright © 2016 McGraw-Hill Education. All rights reserved. No reproduction or distribution without the prior written consent of
McGraw-Hill Education.
76. Disputed Ring. While working in the yard, Tina found a beat up ring. Becca, an eighteen-year old
teenager and neighbor, came over to visit and liked the ring. Tina said, "You can have this old
thing if you would like." Becca replied, "I really like it - Maybe it's a real diamond!" Tina laughingly
told her that there was a one in a billion chance of that and that Tina was more likely to win the
lottery. A few months later Becca ran over to Tina's house and told Tina that the ring was actually
a diamond worth thousands! Becca gave Tina the ring to examine. Tina put it in her pocket and
told Becca that she would never have given it to her if she had realized its value and that
possession was back where it had always belonged. Tina also told Becca that Becca failed
legally to accept the gift because she did not know its true value and also because Tina did not
sign any document turning over title. Becca sues. Which of the following describes the type of gift
at issue?

A. A gift causa mortis


B. A gift inter vivos
C. A gift inter mortis
D. A gift causa vivos
E. A donative gift

77. Disputed Ring. While working in the yard, Tina found a beat up ring. Becca, an eighteen-year old
teenager and neighbor, came over to visit and liked the ring. Tina said, "You can have this old
thing if you would like." Becca replied, "I really like it - Maybe it's a real diamond!" Tina laughingly
told her that there was a one in a billion chance of that and that Tina was more likely to win the
lottery. A few months later Becca ran over to Tina's house and told Tina that the ring was actually
a diamond worth thousands! Becca gave Tina the ring to examine. Tina put it in her pocket and
told Becca that she would never have given it to her if she had realized its value and that
possession was back where it had always belonged. Tina also told Becca that Becca failed
legally to accept the gift because she did not know its true value and also because Tina did not
sign any document turning over title. Becca sues. Which of the following is true regarding Tina's
statement that she was entitled to the ring because she would not have given it to Becca if she
had known the true value?

A. Tina is entitled to ownership of the ring if she can prove that she did not realize its true value.
B. Tina is entitled to ownership of the ring only if she can prove that she did not realize its true
value, and also that there was a difference of at least $1,000 between what she believed the
value to be and its true value.
C. Tina is entitled to ownership of the ring only if she can prove that she did not realize its true
value and also that there was a difference of at least $10,000 between what she believed the
value to be and its true value.
D. Tina is entitled to ownership of the ring but only because neither she nor Becca realized its
true value.
E. Being ignorant of the value of the ring does not entitle Tina to ownership of it.

8-14
Copyright © 2016 McGraw-Hill Education. All rights reserved. No reproduction or distribution without the prior written consent of
McGraw-Hill Education.
78. Disputed Ring. While working in the yard, Tina found a beat up ring. Becca, an eighteen-year old
teenager and neighbor, came over to visit and liked the ring. Tina said, "You can have this old
thing if you would like." Becca replied, "I really like it - Maybe it's a real diamond!" Tina laughingly
told her that there was a one in a billion chance of that and that Tina was more likely to win the
lottery. A few months later Becca ran over to Tina's house and told Tina that the ring was actually
a diamond worth thousands! Becca gave Tina the ring to examine. Tina put it in her pocket and
told Becca that she would never have given it to her if she had realized its value and that
possession was back where it had always belonged. Tina also told Becca that Becca failed
legally to accept the gift because she did not know its true value and also because Tina did not
sign any document turning over title. Becca sues. Which of the following is true regarding Tina's
statement that Becca could not have validly accepted the ring?

A. If the ring is valued at $500 or over, then Tina is correct that Becca did not validly accept the
ring because a writing would have been needed to validate acceptance.
B. If the ring is valued at $1,000 or over, then Tina is correct that Becca did not validly accept the
ring because a writing would have been needed to validate acceptance.
C. If the ring is valued at $10,000 or over, then Tina is correct that Becca did not validly accept
the ring because a writing would have been needed to validate acceptance.
D. Tina is correct that Becca could not have validly accepted the ring when she did not
understand its true value.
E. Tina is incorrect, and Becca validly accepted the ring as a gift.

79. Easement Dispute. Sally buys a house from Bob that borders on a lake. Her deed gives her the
most complete estate a person may have. She is in her bathing suit sunning one day when her
neighbor, Fred cuts through her yard. Sally tells him not to do that anymore. Fred informs her that
he had previously purchased and properly recorded a right to cut across her yard to get lake
access. Sally asks you what she should do to try to get rid of Fred because she doesn't like him
cutting through her yard when she is sunning. What type of estate did Sally purchase from Bob?

A. A life absolute estate


B. A conditional estate
C. A fee simple absolute estate
D. A limited estate
E. A subject estate

80. Easement Dispute. Sally buys a house from Bob that borders on a lake. Her deed gives her the
most complete estate a person may have. She is in her bathing suit sunning one day when her
neighbor, Fred cuts through her yard. Sally tells him not to do that anymore. Fred informs her that
he had previously purchased and properly recorded a right to cut across her yard to get lake
access. Sally asks you what she should do to try to get rid of Fred because she doesn't like him
cutting through her yard when she is sunning. What is the type of right that Fred is exercising by
cutting through the yard called?

A. A profit
B. A license
C. A condition
D. An easement
E. A breach of warranty

8-15
Copyright © 2016 McGraw-Hill Education. All rights reserved. No reproduction or distribution without the prior written consent of
McGraw-Hill Education.
81. Easement Dispute. Sally buys a house from Bob that borders on a lake. Her deed gives her the
most complete estate a person may have. She is in her bathing suit sunning one day when her
neighbor, Fred cuts through her yard. Sally tells him not to do that anymore. Fred informs her that
he had previously purchased and properly recorded a right to cut across her yard to get lake
access. Sally asks you what she should do to try to get rid of Fred because she doesn't like him
cutting through her yard when she is sunning. What is the best advice you can give Sally
regarding getting rid of Fred?

A. To call the police when he shows up because any rights he had, terminated when Sally bought
the property.
B. To sell her property and move on because she has absolutely no hope.
C. To put up with Fred for one year because his interest could only last that long after her
purchase.
D. To put up with Fred for three years because his interest could only last that long after her
purchase.
E. To reach some sort of agreement with him, possibly by paying him to give up the easement.

82. Wedding Photos. Bobby took a number of wedding photos at Jill's wedding. He was paid as the
photographer. On all of the photographs, he appropriately noted in the bottom right-hand corner
the necessary information showing that he was claiming copyright protection. Jill came to see
Bobby three years after the initial photographs were taken and requested that he grant her
permission to run off as many copies as she wanted at the local photo shop from the pictures that
she initially purchased. The photo shop had refused to reproduce the photographs without his
permission. When he refused to give her permission to do so, Jill started a heated argument. She
told Bobby that photographs are not entitled to copyright protection. She also told him that even if
he was correct that there was some copyright protection, she was engaged in fair use; and that,
in any event, damages for copyright infringement are unavailable. Which of the following is
correct regarding Jill's claim that photographs are not subject to copyright protection?

A. She is correct. Photographs are not subject to copyright protection even if taken by a
professional photographer.
B. She is correct but only because family pictures are involved. Family pictures may not be the
subject of copyright, but landscape photographs may be the subject of copyright protection.
C. She is partially correct. Bobby was entitled to copyright protection on the first picture. After Jill
purchased the first picture, however, she could make as many copies as she wanted.
D. She is correct only because Bobby had not registered the photographs for copyright
protection.
E. She is incorrect.

8-16
Copyright © 2016 McGraw-Hill Education. All rights reserved. No reproduction or distribution without the prior written consent of
McGraw-Hill Education.
83. Wedding Photos. Bobby took a number of wedding photos at Jill's wedding. He was paid as the
photographer. On all of the photographs, he appropriately noted in the bottom right-hand corner
the necessary information showing that he was claiming copyright protection. Jill came to see
Bobby three years after the initial photographs were taken and requested that he grant her
permission to run off as many copies as she wanted at the local photo shop from the pictures that
she initially purchased. The photo shop had refused to reproduce the photographs without his
permission. When he refused to give her permission to do so, Jill started a heated argument. She
told Bobby that photographs are not entitled to copyright protection. She also told him that even if
he was correct that there was some copyright protection, she was engaged in fair use; and that,
in any event, damages for copyright infringement are unavailable. Which of the following is true
regarding Jill's claim that she was engaged in fair use?

A. It is likely that she would win on the claim because she does not seek to use the photographs
in a business pursuit.
B. It is likely that she would win on the claim because she only seeks to copy what she has
already purchased.
C. This issue would not come up because Bobby is not entitled to copyright protection on the
photographs.
D. She would lose because the fair-use doctrine is only available to educators.
E. She would likely lose under the weighing test mandated by the Copyright Act.

84. Wedding Photos. Bobby took a number of wedding photos at Jill's wedding. He was paid as the
photographer. On all of the photographs, he appropriately noted in the bottom right-hand corner
the necessary information showing that he was claiming copyright protection. Jill came to see
Bobby three years after the initial photographs were taken and requested that he grant her
permission to run off as many copies as she wanted at the local photo shop from the pictures that
she initially purchased. The photo shop had refused to reproduce the photographs without his
permission. When he refused to give her permission to do so, Jill started a heated argument. She
told Bobby that photographs are not entitled to copyright protection. She also told him that even if
he was correct that there was some copyright protection, she was engaged in fair use; and that,
in any event, damages for copyright infringement are unavailable. Which of the following is
correct regarding Jill's claim that damages for copyright infringement are unavailable?

A. Damages are available, but a copyrighted work must be registered in order for the creator to
recover damages from infringement.
B. It is not necessary that a copyrighted work be registered in order for the creator to obtain
damages.
C. Damages for copyright infringement are only available if two businesses are involved, not in
disputes involving an individual such as Jill.
D. Some material that is subject to copyright must be registered before its creator may recover
damages for infringement, but that is not true for photographs.
E. She is correct that damages are unavailable in copyright infringement actions because only
injunctive relief is available.

8-17
Copyright © 2016 McGraw-Hill Education. All rights reserved. No reproduction or distribution without the prior written consent of
McGraw-Hill Education.
85. Wedding Photos. Bobby took a number of wedding photos at Jill's wedding. He was paid as the
photographer. On all of the photographs, he appropriately noted in the bottom right-hand corner
the necessary information showing that he was claiming copyright protection. Jill came to see
Bobby three years after the initial photographs were taken and requested that he grant her
permission to run off as many copies as she wanted at the local photo shop from the pictures that
she initially purchased. The photo shop had refused to reproduce the photographs without his
permission. When he refused to give her permission to do so, Jill started a heated argument. She
told Bobby that photographs are not entitled to copyright protection. She also told him that even if
he was correct that there was some copyright protection, she was engaged in fair use; and that,
in any event, damages for copyright infringement are unavailable. If Bobby decides to register the
photographs under copyright law, how would he go about doing so?

A. He would register by filing a form with the Register of Copyright and providing two copies of
the copyrighted materials to the Library of Congress.
B. He would register by filing a form with Register of Copyright only.
C. He would register by providing two copies of the copyrighted materials to the Library of
Congress only.
D. He would register by affixing the appropriate symbol at the bottom of the photograph followed
by the first date of publication and his name.
E. There is nothing he can do because the photographs are not subject to copyright protection.

86. Scuba Diving. Marcy invented a new type of mask for scuba divers that was not subject to
fogging. She agrees to allow Jenny to manufacture and sell the mask. She receives a sum of
money for every mask that Jenny sells. Marcy also entered into an agreement with Frank to allow
him to sell the masks but only if he also purchased non-patented diving suits from Marcy. All
parties proceeded to do very well with their sales.
Marcy's agreement with Jenny allowing Jenny to sell the mask is referred to as which of the
following?

A. A license
B. A patent agreement
C. A trade agreement
D. An illegal agreement
E. A franchise agreement

87. Scuba Diving. Marcy invented a new type of mask for scuba divers that was not subject to
fogging. She agrees to allow Jenny to manufacture and sell the mask. She receives a sum of
money for every mask that Jenny sells. Marcy also entered into an agreement with Frank to allow
him to sell the masks but only if he also purchased non-patented diving suits from Marcy. All
parties proceeded to do very well with their sales.
Payments that Marcy would receive from Jenny for the sale of the mask are referred to as which
of the following?

A. Profits
B. Receipts
C. Royalties
D. Payoffs
E. Illegal

8-18
Copyright © 2016 McGraw-Hill Education. All rights reserved. No reproduction or distribution without the prior written consent of
McGraw-Hill Education.
88. Scuba Diving. Marcy invented a new type of mask for scuba divers that was not subject to
fogging. She agrees to allow Jenny to manufacture and sell the mask. She receives a sum of
money for every mask that Jenny sells. Marcy also entered into an agreement with Frank to allow
him to sell the masks but only if he also purchased non-patented diving suits from Marcy. All
parties proceeded to do very well with their sales.
Which of the following describes the agreement between Marcy and Frank?

A. It is a legal tying arrangement.


B. It is a legal cross-licensing agreement.
C. It is an illegal tying arrangement.
D. It is an illegal cross-licensing agreement.
E. It is both a legal tying and a legal cross-licensing agreement.

89. Lottery Winnings. Frank, a hypochondriac who was also very compulsive, was having minor
surgery to repair a bone spur on his foot. He had just purchased a lottery ticket for a chance at
the grand prize of $30,000,000. Frank's girlfriend, Bubbles, went with him to the hospital. While in
the waiting room, Frank said to her, "Bubbles, I may not make it out of this bone spur surgery.
Take my lottery ticket. If I don't make it, I hope you win and live it up, but please don't get another
boyfriend." Bubbles replied, "I could never be happy without you." A nurse saw and heard the
whole exchange. Frank came out of the surgery just fine but with a sore foot. While he was
recuperating, that evening, Bubbles watched the lottery drawing and discovered that Frank's
ticket was indeed the winning ticket. She immediately moved out and collected the winnings.
Frank saw her on television with her new boyfriend, George. She appeared to be very happy. He
checked the numbers and discovered that she won off of his ticket. Frank says that the lottery
money is his. Which of the following describes the type of gift at issue?

A. A gift causa mortis


B. A gift inter vivos
C. A gift inter mortis
D. A gift causa vivos
E. A donative gift

8-19
Copyright © 2016 McGraw-Hill Education. All rights reserved. No reproduction or distribution without the prior written consent of
McGraw-Hill Education.
90. Lottery Winnings. Frank, a hypochondriac who was also very compulsive, was having minor
surgery to repair a bone spur on his foot. He had just purchased a lottery ticket for a chance at
the grand prize of $30,000,000. Frank's girlfriend, Bubbles, went with him to the hospital. While in
the waiting room, Frank said to her, "Bubbles, I may not make it out of this bone spur surgery.
Take my lottery ticket. If I don't make it, I hope you win and live it up, but please don't get another
boyfriend." Bubbles replied, "I could never be happy without you." A nurse saw and heard the
whole exchange. Frank came out of the surgery just fine but with a sore foot. While he was
recuperating, that evening, Bubbles watched the lottery drawing and discovered that Frank's
ticket was indeed the winning ticket. She immediately moved out and collected the winnings.
Frank saw her on television with her new boyfriend, George. She appeared to be very happy. He
checked the numbers and discovered that she won off of his ticket. Frank says that the lottery
money is his. Which of the following is true regarding Frank's statement at the hospital regarding
Bubbles taking the ticket?

A. At that point he made a valid gift that could not be revoked.


B. He did not make a gift because he placed a condition on it.
C. He made an irrevocable gift at that time only if Bubbles never had another boyfriend;
otherwise, she had to give the ticket and any resulting cash to Frank.
D. He made an irrevocable gift at that time if Bubbles refrained from having another boyfriend
until she cashed the ticket; and, after the ticket was converted, the condition no longer applied.
E. None of these.

91. Lottery Winnings. Frank, a hypochondriac who was also very compulsive, was having minor
surgery to repair a bone spur on his foot. He had just purchased a lottery ticket for a chance at
the grand prize of $30,000,000. Frank's girlfriend, Bubbles, went with him to the hospital. While in
the waiting room, Frank said to her, "Bubbles, I may not make it out of this bone spur surgery.
Take my lottery ticket. If I don't make it, I hope you win and live it up, but please don't get another
boyfriend." Bubbles replied, "I could never be happy without you." A nurse saw and heard the
whole exchange. Frank came out of the surgery just fine but with a sore foot. While he was
recuperating, that evening, Bubbles watched the lottery drawing and discovered that Frank's
ticket was indeed the winning ticket. She immediately moved out and collected the winnings.
Frank saw her on television with her new boyfriend, George. She appeared to be very happy. He
checked the numbers and discovered that she won off of his ticket. Frank says that the lottery
money is his. Which of the following is true regarding rightful ownership of the lottery money?

A. Frank's gift was automatically revoked when he recovered, and the lottery ticket and any
proceeds are validly his.
B. Although the gift was not automatically revoked on his recovery, Frank has the right to revoke
the gift and recover the lottery proceeds.
C. The gift was not automatically revoked on his recovery and since Frank did not revoke the gift
prior to Bubbles cashing the ticket, he has no rights to any funds.
D. Frank has the right to all funds only because Bubbles has a new boyfriend.
E. Frank has no rights because once he gave the ticket to Bubbles, he gave up all his rights to it.

Essay Questions

8-20
Copyright © 2016 McGraw-Hill Education. All rights reserved. No reproduction or distribution without the prior written consent of
McGraw-Hill Education.
92. Discuss the extent of ownership rights when a person owns a fee simple absolute in land,
meaning what interests, if any, does the owner have in addition to rights to use the surface of the
property.

93. Identify where a trademark is registered, what one should do when requesting registration and
what the governmental office involved would then do. Identify circumstances under which a
trademark will not be accepted for registration.

94. Ellen, who runs a successful nail salon, ran across a very interesting pamphlet on customer
relations. The pamphlet was fifty pages long and cost $70.00. Ellen thought the price was
outrageous. Accordingly, she bought one copy and photocopied the pages that she thought were
pertinent and gave them to her employees. She copied forty of the pages out of the pamphlet for
each of twenty employees. A disgruntled employee informed the publisher of the pamphlet what
Ellen had done and she was charged with copyright infringement. Ellen defended on the basis of
the fair-use doctrine. List the four factors a court would consider in determining whether a
violation occurred and discuss whether or not you think Ellen would prevail and why.

8-21
Copyright © 2016 McGraw-Hill Education. All rights reserved. No reproduction or distribution without the prior written consent of
McGraw-Hill Education.
95. What three criteria must be satisfied in order to obtain a patent that is not a design patent?

96. Cindy sold her old used automobile to Bruce for $1,000. Cindy told Bruce to come and take the
car and that the transfer would be complete. Is Cindy correct, and why or why not?

97. Bernice enjoys snooping, particularly through her neighbor's trash. She sneaks out and grabs the
neighbor's trash after they have gone to work but before the trash collector comes. She goes
through it to find out confidential information regarding the neighbors, but never tells anyone else
or uses the information for any purpose. Discuss whether you believe it is legal for Bernice to go
through the neighbor's trash and why or why not. Also, discuss whether you believe it is ethical
for Bernice to go through the neighbor's trash, and why or why not.

8-22
Copyright © 2016 McGraw-Hill Education. All rights reserved. No reproduction or distribution without the prior written consent of
McGraw-Hill Education.
Chapter 08 Real, Personal, and Intellectual Property Answer Key

True / False Questions

1. Real property includes land and everything permanently attached to it.

TRUE

Real property includes land and everything permanently attached to it.

AACSB: Analytical Thinking


Accessibility: Keyboard Navigation
Blooms: Remember
Difficulty: 1 Easy
Learning Objective: 08-01 How are the topics of real; personal; and intellectual property related?
Topic: The Nature of Real, Personal, and Intellectual Property

2. The laws of intellectual property protect tangible property.

FALSE

The laws of intellectual property protect intangible property that is primarily the result of mental
creativity rather than physical effort.

AACSB: Analytical Thinking


Accessibility: Keyboard Navigation
Blooms: Understand
Difficulty: 2 Medium
Learning Objective: 08-01 How are the topics of real; personal; and intellectual property related?
Topic: The Nature of Real, Personal, and Intellectual Property

3. Ownership of real property includes the airspace above the land but not the mineral rights
below the land.

FALSE

Ownership of real property includes the airspace above the land and the mineral rights below
the land.

AACSB: Analytical Thinking


Accessibility: Keyboard Navigation
Blooms: Understand
Difficulty: 2 Medium
Learning Objective: 08-02 How does real property law balance private and public rights?
Topic: Real Property

8-23
Copyright © 2016 McGraw-Hill Education. All rights reserved. No reproduction or distribution without the prior written consent of
McGraw-Hill Education.
4. A landowner may be required to involuntarily transfer their property to the government.

TRUE

A landowner may be required to involuntarily transfer their property to the government for the
benefit of the public and to protect public health, safety, and welfare.

AACSB: Analytical Thinking


Accessibility: Keyboard Navigation
Blooms: Understand
Difficulty: 2 Medium
Learning Objective: 08-01 How are the topics of real; personal; and intellectual property related?
Topic: The Nature of Real, Personal, and Intellectual Property

5. An easement is a revocable right to use some part of another's land for a specific purpose
without taking anything from it.

TRUE

An easement is an irrevocable right to use some part of another's land for a specific purpose
without taking anything from it.

AACSB: Analytical Thinking


Accessibility: Keyboard Navigation
Blooms: Remember
Difficulty: 1 Easy
Learning Objective: 08-03 What are the interests in real property that one can hold?
Topic: Real Property

6. Subleasing of leased property by the tenant to another party is permissible unless specifically
prohibited by the lease.

TRUE

Subleasing of the property by the tenant to another party is permissible unless specifically
prohibited by the lease.

AACSB: Analytical Thinking


Accessibility: Keyboard Navigation
Blooms: Understand
Difficulty: 2 Medium
Learning Objective: 08-03 What are the interests in real property that one can hold?
Topic: Real Property

8-24
Copyright © 2016 McGraw-Hill Education. All rights reserved. No reproduction or distribution without the prior written consent of
McGraw-Hill Education.
7. A license is a right to use another's property that is permanent and revocable.

FALSE

A license is a right to use another's property that is both temporary and revocable.

AACSB: Analytical Thinking


Accessibility: Keyboard Navigation
Blooms: Remember
Difficulty: 1 Easy
Learning Objective: 08-03 What are the interests in real property that one can hold?
Topic: Real Property

8. Condemnation is a legal process that is protected by the federal constitution and allows the
government to take private property only if the property is used for new schools or roads.

FALSE

Condemnation is a legal process that is protected by the Fifth Amendment and allows the
involuntary transfer of private property for public good. It may be exercised on behalf of a
private company operating to benefit the public.

AACSB: Analytical Thinking


Accessibility: Keyboard Navigation
Blooms: Understand
Difficulty: 2 Medium
Learning Objective: 08-03 What are the interests in real property that one can hold?
Topic: Real Property

9. All property that is not land or permanently affixed to land is personal property.

TRUE

All property that is not land or permanently affixed to land is personal property.

AACSB: Analytical Thinking


Accessibility: Keyboard Navigation
Blooms: Remember
Difficulty: 1 Easy
Learning Objective: 08-04 What are the types of personal property?
Topic: Personal Property

10. A book is typically considered intellectual property.

FALSE

A book is typically considered personal property.

AACSB: Analytical Thinking


Accessibility: Keyboard Navigation
Blooms: Understand
Difficulty: 2 Medium
Learning Objective: 08-04 What are the types of personal property?
Topic: Personal Property

8-25
Copyright © 2016 McGraw-Hill Education. All rights reserved. No reproduction or distribution without the prior written consent of
McGraw-Hill Education.
11. Personal property can be tangible property but not intangible property.

FALSE

Personal property can be both tangible property and intangible property.

AACSB: Analytical Thinking


Accessibility: Keyboard Navigation
Blooms: Understand
Difficulty: 2 Medium
Learning Objective: 08-04 What are the types of personal property?
Topic: Personal Property

12. A causa mortis gift may be revoked any time before the donor's death and is automatically
revoked if the donor recovers.

TRUE

A causa mortis gift may be revoked any time before the donor's death and is automatically
revoked if the donor recovers.

AACSB: Analytical Thinking


Accessibility: Keyboard Navigation
Blooms: Remember
Difficulty: 1 Easy
Learning Objective: 08-05 How do you transfer personal property?
Topic: Personal Property

13. To register a domain name on the Internet, an application must be filed with Network Solutions
Inc., which is funded by the National Science Foundation.

TRUE

Network Solutions Inc. (NSI), which is funded by the National Science Foundation, is
responsible for registering domain names.

AACSB: Technology
Accessibility: Keyboard Navigation
Blooms: Remember
Difficulty: 1 Easy
Learning Objective: 08-07 What are the similarities and differences among the methods for protecting various types of
intellectual property?
Topic: Intellectual Property

8-26
Copyright © 2016 McGraw-Hill Education. All rights reserved. No reproduction or distribution without the prior written consent of
McGraw-Hill Education.
14. A person who applies for a domain name on the Internet will be granted the domain name so
long as no one else holds that name.

FALSE

Anyone seeking to register a domain name must now state in the application that the name
will not infringe on anyone else's intellectual property rights and that the registrant intends to
use the name on a regular basis on the Internet.

AACSB: Technology
Accessibility: Keyboard Navigation
Blooms: Understand
Difficulty: 2 Medium
Learning Objective: 08-07 What are the similarities and differences among the methods for protecting various types of
intellectual property?
Topic: Intellectual Property

15. Trade dress refers to the overall appearance and image of a product and, is entitled to the
same protection as a trademark.

TRUE

Trade dress refers to the overall appearance and image of a product and is entitled to the
same protection as a trademark.

AACSB: Analytical Thinking


Accessibility: Keyboard Navigation
Blooms: Understand
Difficulty: 2 Medium
Learning Objective: 08-07 What are the similarities and differences among the methods for protecting various types of
intellectual property?
Topic: Intellectual Property

16. Copyrights protect the ideas of creative ideas.

FALSE

Copyrights protect the expression of creative ideas, but not the ideas themselves.

AACSB: Analytical Thinking


Accessibility: Keyboard Navigation
Blooms: Remember
Difficulty: 1 Easy
Learning Objective: 08-07 What are the similarities and differences among the methods for protecting various types of
intellectual property?
Topic: Intellectual Property

8-27
Copyright © 2016 McGraw-Hill Education. All rights reserved. No reproduction or distribution without the prior written consent of
McGraw-Hill Education.
17. Copyright is the protection of the expression of a fixed form that expresses ideas.

TRUE

Copyrights are the protection of the fixed form that expresses the ideas.

AACSB: Analytical Thinking


Accessibility: Keyboard Navigation
Blooms: Remember
Difficulty: 2 Medium
Learning Objective: 08-07 What are the similarities and differences among the methods for protecting various types of
intellectual property?
Topic: Intellectual Property

18. When copyright is at issue, proof that an item is similar is not necessarily infringement.

TRUE

It is not always easy to determine whether a copyright has been infringed, even when two
works are similar.

AACSB: Analytical Thinking


Accessibility: Keyboard Navigation
Blooms: Understand
Difficulty: 2 Medium
Learning Objective: 08-07 What are the similarities and differences among the methods for protecting various types of
intellectual property?
Topic: Intellectual Property

19. A teacher cannot be held liable for copyright infringement under the Fair-Use Doctrine so long
as copies are only used for educational purposes.

FALSE

In determining whether the fair-use doctrine provides a valid defense to a claim of copyright
infringement, Section 107 of the Copyright Act requires that the court weigh the following four
factors: the purpose and character of the use, the nature of the copyrighted work, the amount
and substantiality of the portion used in relation to the copyrighted work as a whole, and the
effect of the use on the potential market for or value of the copyrighted work.

AACSB: Reflective Thinking


Accessibility: Keyboard Navigation
Blooms: Understand
Difficulty: 2 Medium
Learning Objective: 08-07 What are the similarities and differences among the methods for protecting various types of
intellectual property?
Topic: Intellectual Property

8-28
Copyright © 2016 McGraw-Hill Education. All rights reserved. No reproduction or distribution without the prior written consent of
McGraw-Hill Education.
20. Tying arrangements are an illegal agreement in which the sale of one product is linked to the
sale of another.

TRUE

A tying arrangement is an illegal agreement in which the sale of one product is linked to the
sale of another.

AACSB: Analytical Thinking


Accessibility: Keyboard Navigation
Blooms: Remember
Difficulty: 1 Easy
Learning Objective: 08-07 What are the similarities and differences among the methods for protecting various types of
intellectual property?
Topic: Intellectual Property

21. A patent grants the holder exclusive rights for 40 years.

FALSE

A patent grants the holder exclusive rights for 20 years.

AACSB: Analytical Thinking


Accessibility: Keyboard Navigation
Blooms: Remember
Difficulty: 1 Easy
Learning Objective: 08-07 What are the similarities and differences among the methods for protecting various types of
intellectual property?
Topic: Intellectual Property

22. An invention may be considered a trade secret.

TRUE

Inventions and designs may be considered trade secrets.

AACSB: Analytical Thinking


Accessibility: Keyboard Navigation
Blooms: Understand
Difficulty: 2 Medium
Learning Objective: 08-07 What are the similarities and differences among the methods for protecting various types of
intellectual property?
Topic: Intellectual Property

8-29
Copyright © 2016 McGraw-Hill Education. All rights reserved. No reproduction or distribution without the prior written consent of
McGraw-Hill Education.
23. Trade-secret protection is lost if another business discovers the trade secret in a lawful
manner.

TRUE

Trade-secret protection is lost once someone discovers the secret lawfully.

AACSB: Analytical Thinking


Accessibility: Keyboard Navigation
Blooms: Understand
Difficulty: 2 Medium
Learning Objective: 08-07 What are the similarities and differences among the methods for protecting various types of
intellectual property?
Topic: Intellectual Property

24. In Germany land expropriated by the government reverts to its former owner if the government
no longer needs the property.

TRUE

In Germany, if the property is no longer needed by the German government, it must revert to
its former owner.

AACSB: Analytical Thinking


Accessibility: Keyboard Navigation
Blooms: Remember
Difficulty: 1 Easy
Learning Objective: 08-03 What are the interests in real property that one can hold?
Topic: Real Property

25. In Kelo v. City of New London, the U.S. Supreme Court ruled that as a matter of law, the
government violates the Fifth Amendment by taking private property and selling it for private
development.

FALSE

In Kelo v. City of New London the U.S. Supreme Court ruled that a government's taking of
private property to sell for private development may qualify as a "public use" within the takings
clause.

AACSB: Analytical Thinking


Accessibility: Keyboard Navigation
Blooms: Understand
Difficulty: 2 Medium
Learning Objective: 08-03 What are the interests in real property that one can hold?
Topic: Real Property

8-30
Copyright © 2016 McGraw-Hill Education. All rights reserved. No reproduction or distribution without the prior written consent of
McGraw-Hill Education.
26. Lost property and mislaid property are the same legal theories.

FALSE

Mislaid property differs from lost property in that the owner has intentionally placed the
property somewhere but has forgotten its location.

AACSB: Analytical Thinking


Accessibility: Keyboard Navigation
Blooms: Understand
Difficulty: 2 Medium
Learning Objective: 08-05 How do you transfer personal property?
Topic: Personal Property

Multiple Choice Questions

27. What are the three basic types of property?

A. Real, absolute, and personal


B. Real, intellectual, and land
C. Intellectual, land, and minerals
D. Real, personal, and intellectual
E. None of these choices.

The three basis types of property are real, personal, and intellectual.

AACSB: Analytical Thinking


Accessibility: Keyboard Navigation
Blooms: Remember
Difficulty: 1 Easy
Learning Objective: 08-01 How are the topics of real; personal; and intellectual property related?
Topic: The Nature of Real, Personal, and Intellectual Property

8-31
Copyright © 2016 McGraw-Hill Education. All rights reserved. No reproduction or distribution without the prior written consent of
McGraw-Hill Education.
28. Land and everything permanently attached to it are known as:

A. Real property
B. Personal property
C. Absolute property
D. Listed property
E. Fee property

Real property, commonly referred to as realty, is land and everything permanently attached to
it.

AACSB: Analytical Thinking


Accessibility: Keyboard Navigation
Blooms: Remember
Difficulty: 1 Easy
Learning Objective: 08-01 How are the topics of real; personal; and intellectual property related?
Topic: The Nature of Real, Personal, and Intellectual Property

29. The right to enter onto property to remove underground materials is included in the ownership
of ____________________.

A. subsurface rights
B. underground rights
C. earth rights
D. terrestrial rights
E. None of these, because there is no such right

Ownership of subsurface rights includes the right to enter onto the property to remove the
underground materials.

AACSB: Analytical Thinking


Accessibility: Keyboard Navigation
Blooms: Remember
Difficulty: 1 Easy
Learning Objective: 08-02 How does real property law balance private and public rights?
Topic: Real Property

8-32
Copyright © 2016 McGraw-Hill Education. All rights reserved. No reproduction or distribution without the prior written consent of
McGraw-Hill Education.
30. The duration of one's ownership interest and the power one has over using the land depends
on the type of ______ one is said to hold.

A. condition
B. estate
C. tenancy
D. hold
E. acknowledgement

The duration of one's ownership interest and the power one has over using the land depend
on the type of estate one is said to hold.

AACSB: Analytical Thinking


Accessibility: Keyboard Navigation
Blooms: Understand
Difficulty: 2 Medium
Learning Objective: 08-02 How does real property law balance private and public rights?
Topic: Real Property

31. Real property ownership includes airspace above the land, water rights flowing across or
beneath the land, and ___________________.

A. trees growing next to the land


B. the right to divert the water to deprive landowners downstream from use of the water
C. walkways leading to the land
D. mineral rights under the land
E. roadways providing access to the land

Real property ownership extends to what lies beneath the land.

AACSB: Analytical Thinking


Accessibility: Keyboard Navigation
Blooms: Understand
Difficulty: 2 Medium
Learning Objective: 08-02 How does real property law balance private and public rights?
Topic: Real Property

8-33
Copyright © 2016 McGraw-Hill Education. All rights reserved. No reproduction or distribution without the prior written consent of
McGraw-Hill Education.
32. A ______ estate is the most complete estate a person may have.

A. conditional estate
B. life estate
C. leasehold estate
D. future interest
E. fee simple absolute

With regard to estates in land, a fee simple absolute is the most complete estate a person may
have.

AACSB: Analytical Thinking


Accessibility: Keyboard Navigation
Blooms: Remember
Difficulty: 1 Easy
Learning Objective: 08-03 What are the interests in real property that one can hold?
Topic: Real Property

33. The owner of a ______ estate possesses the same interest as the owner of a fee simple
absolute, only this interest is subject to a condition that may result in termination of the
interest.

A. conditional estate
B. life estate
C. leasehold estate
D. future interest
E. fee simple absolute

The owner of a conditional estate possesses the same interest as the owner of a fee simple
absolute except that this interest is subject to a condition: Should a certain required event fail
to occur, or should a specific prohibited event occur, the interest will be terminated.

AACSB: Analytical Thinking


Accessibility: Keyboard Navigation
Blooms: Understand
Difficulty: 2 Medium
Learning Objective: 08-03 What are the interests in real property that one can hold?
Topic: Real Property

8-34
Copyright © 2016 McGraw-Hill Education. All rights reserved. No reproduction or distribution without the prior written consent of
McGraw-Hill Education.
34. An ownership interest in which the holder has the right to possess the property until his or her
death is considered a ___________.

A. conditional estate
B. life estate
C. leasehold estate
D. future interest
E. fee simple absolute

An ownership interest in which the holder has the right to possess the property until his or her
death is considered a life estate.

AACSB: Analytical Thinking


Accessibility: Keyboard Navigation
Blooms: Remember
Difficulty: 1 Easy
Learning Objective: 08-03 What are the interests in real property that one can hold?
Topic: Real Property

35. A ______ is a person's present right to future property ownership and possession.

A. conditional estate
B. life estate
C. leasehold estate
D. future interest
E. fee simple absolute

A future interest is a person's present right to property ownership and possession in the future.

AACSB: Analytical Thinking


Accessibility: Keyboard Navigation
Blooms: Remember
Difficulty: 1 Easy
Learning Objective: 08-03 What are the interests in real property that one can hold?
Topic: Real Property

8-35
Copyright © 2016 McGraw-Hill Education. All rights reserved. No reproduction or distribution without the prior written consent of
McGraw-Hill Education.
36. An owner of a ______ has a possessory interest but not an ownership interest.

A. conditional estate
B. life estate
C. leasehold
D. future interest
E. fee simple absolute

The interest of a leasehold differs from the interests previously described in that the holder of
such an estate has a possessory interest but not an ownership interest.

AACSB: Analytical Thinking


Accessibility: Keyboard Navigation
Blooms: Understand
Difficulty: 2 Medium
Learning Objective: 08-03 What are the interests in real property that one can hold?
Topic: Real Property

37. Which of the following is false regarding property interests in Vietnam?

A. The constitution asserts that the state owns all the land.
B. If an individual wants to use land, he or she must pay tax on it as a form of rent.
C. Transference of property can occur only with the approval of a state official.
D. A new owner can never be given a longer term of right or more extensive rights over the
land than the original owner had.
E. The transferring owner determines the price for which property will be transferred.

A state official determines the price at which the property will be transferred.

AACSB: Diversity
Accessibility: Keyboard Navigation
Blooms: Remember
Difficulty: 1 Easy
Learning Objective: 08-03 What are the interests in real property that one can hold?
Topic: Real Property

8-36
Copyright © 2016 McGraw-Hill Education. All rights reserved. No reproduction or distribution without the prior written consent of
McGraw-Hill Education.
38. Which of the following are estates that do not include the right to possess the property?

A. Easements
B. Profits
C. Licenses
D. Easements, profits, and licenses
E. Easements and profits, but not licenses

Estates that do not include the right to possess the property include easements, profits, and
licenses.

AACSB: Analytical Thinking


Accessibility: Keyboard Navigation
Blooms: Understand
Difficulty: 2 Medium
Learning Objective: 08-03 What are the interests in real property that one can hold?
Topic: Real Property

39. A(n) ______ is an irrevocable right to use some part of another's land for a specific purpose
without taking anything from it.

A. appurtenance
B. profit
C. easement
D. fixation
E. joint use

An easement is an irrevocable right to use some part of another's land for a specific purpose
without taking anything from it.

AACSB: Analytical Thinking


Accessibility: Keyboard Navigation
Blooms: Understand
Difficulty: 2 Medium
Learning Objective: 08-03 What are the interests in real property that one can hold?
Topic: Real Property

8-37
Copyright © 2016 McGraw-Hill Education. All rights reserved. No reproduction or distribution without the prior written consent of
McGraw-Hill Education.
40. A(n) ______ is the right to go onto someone's land and take part of the land or a product of it
away from the land.

A. appurtenance
B. profit
C. easement
D. fixation
E. joint use

A profit is the right to go onto someone's land and take part of the land or a product of it away
from the land.

AACSB: Analytical Thinking


Accessibility: Keyboard Navigation
Blooms: Understand
Difficulty: 2 Medium
Learning Objective: 08-03 What are the interests in real property that one can hold?
Topic: Real Property

41. Which of the following procedures must be followed in order to effectuate a voluntary transfer
of real property?

A. Execution
B. Delivery
C. Acceptance
D. Execution, delivery, acceptance, and recording
E. Execution and delivery, but not acceptance

To effectuate a voluntary transfer, the owner must follow certain legal procedures, namely;
execution, delivery, acceptance, and recording.

AACSB: Analytical Thinking


Accessibility: Keyboard Navigation
Blooms: Understand
Difficulty: 2 Medium
Learning Objective: 08-03 What are the interests in real property that one can hold?
Topic: Real Property

8-38
Copyright © 2016 McGraw-Hill Education. All rights reserved. No reproduction or distribution without the prior written consent of
McGraw-Hill Education.
42. Which of the following is the legal process by which a transfer of property is made by a
governmental entity against the protest of the property owner pursuant to the takings clause of
the Fifth Amendment?

A. Allocation
B. Appropriation
C. Condemnation
D. Substitution
E. Publication

Condemnation is the legal process by which a transfer of property is made against the protest
of the property owner pursuant to the takings clause of the Fifth Amendment.

AACSB: Analytical Thinking


Accessibility: Keyboard Navigation
Blooms: Remember
Difficulty: 1 Easy
Learning Objective: 08-03 What are the interests in real property that one can hold?
Topic: Real Property

43. Which of the following consists of the fruits of one's mind?

A. Theoretical property
B. Cognitive property
C. Intellectual property
D. Protected property
E. Tradable property

Intellectual property consists of the fruits of one's mind.

AACSB: Analytical Thinking


Accessibility: Keyboard Navigation
Blooms: Remember
Difficulty: 1 Easy
Learning Objective: 08-01 How are the topics of real; personal; and intellectual property related?
Topic: The Nature of Real, Personal, and Intellectual Property

8-39
Copyright © 2016 McGraw-Hill Education. All rights reserved. No reproduction or distribution without the prior written consent of
McGraw-Hill Education.
44. An example of tangible property is

A. Furniture and cars


B. Furniture but not cars
C. Bank accounts
D. Bank accounts and stocks
E. Stocks and insurance policies

An example of tangible property is furniture and cars.

AACSB: Analytical Thinking


Accessibility: Keyboard Navigation
Blooms: Understand
Difficulty: 2 Medium
Learning Objective: 08-04 What are the types of personal property?
Topic: Personal Property

45. What type of gift is made by a person during his or her lifetime?

A. Adverse possession
B. Profit
C. An easement
D. Causa mortis
E. Inter vivos

An inter vivos gift is a gift that is made by a person during his or her lifetime.

AACSB: Analytical Thinking


Accessibility: Keyboard Navigation
Blooms: Remember
Difficulty: 1 Easy
Learning Objective: 08-05 How do you transfer personal property?
Topic: Personal Property

46. What is the term for a gift that is made in contemplation of one's immediate death?

A. Adverse possession
B. Profit
C. An easement
D. Causa mortis
E. Inter vivos

Causa mortis is a gift that is made in contemplation of one's immediate death.

AACSB: Analytical Thinking


Accessibility: Keyboard Navigation
Blooms: Remember
Difficulty: 1 Easy
Learning Objective: 08-05 How do you transfer personal property?
Topic: Personal Property

8-40
Copyright © 2016 McGraw-Hill Education. All rights reserved. No reproduction or distribution without the prior written consent of
McGraw-Hill Education.
47. A(n) ________ is a distinctive mark, word, design, picture, or arrangement that is used by a
producer in conjunction with a product and tends to cause consumers to identify the product
with the producer.

A. copyright
B. patent
C. trade secret
D. trademark
E. intellamark

A trademark is a distinctive mark, word, design, picture, or arrangement that is used by a


producer in conjunction with a product and tends to cause consumers to identify the product
with the producer.

AACSB: Analytical Thinking


Accessibility: Keyboard Navigation
Blooms: Remember
Difficulty: 1 Easy
Learning Objective: 08-07 What are the similarities and differences among the methods for protecting various types of
intellectual property?
Topic: Intellectual Property

48. Which of the following is true regarding whether the shape of a product or package may be a
trademark?

A. The shape of a product may be a trademark if it is nonfunctional, but the shape of a


package may not be the subject of a trademark.
B. The shape of a product may be the subject of a trademark if it is functional, but the shape
of the package may not be the subject of a trademark.
C. The shape of a product or package may be a trademark if it is functional.
D. The shape of a product or the shape of a package may be a trademark if it is
nonfunctional.
E. The shape of a package may be a trademark if it is nonfunctional, but the shape of a
product may not be the subject or a trademark.

The shape of a product or package may be a trademark if it is nonfunctional.

AACSB: Reflective Thinking


Accessibility: Keyboard Navigation
Blooms: Understand
Difficulty: 2 Medium
Learning Objective: 08-07 What are the similarities and differences among the methods for protecting various types of
intellectual property?
Topic: Intellectual Property

8-41
Copyright © 2016 McGraw-Hill Education. All rights reserved. No reproduction or distribution without the prior written consent of
McGraw-Hill Education.
49. Under which of the following is a trademark protected under state common law?

A. If it is used interstate
B. If it is used intrastate
C. If it is used commercially
D. If it is used intrastate and commercially
E. A trademark cannot be protected under state common law

A trademark used intrastate is protected under state common law.

AACSB: Analytical Thinking


Accessibility: Keyboard Navigation
Blooms: Understand
Difficulty: 2 Medium
Learning Objective: 08-07 What are the similarities and differences among the methods for protecting various types of
intellectual property?
Topic: Intellectual Property

50. To be protected in ____________ use, a trademark must be registered with the U.S. Patent
Office under the Lanham Act of 1947.

A. interstate
B. intrastate
C. interstate and intrastate
D. commercial
E. interstate, intrastate, and commercial

To be protected in interstate use, the trademark must be registered with the U.S. Patent Office
under the Lanham Act of 1947.

AACSB: Analytical Thinking


Accessibility: Keyboard Navigation
Blooms: Remember
Difficulty: 1 Easy
Learning Objective: 08-07 What are the similarities and differences among the methods for protecting various types of
intellectual property?
Topic: Intellectual Property

8-42
Copyright © 2016 McGraw-Hill Education. All rights reserved. No reproduction or distribution without the prior written consent of
McGraw-Hill Education.
51. If a trademark is registered, what may the owner obtain in the event of infringement from a
person who used the trademark to pass off goods as being those of the mark owner?

A. Damages
B. An injunction prohibiting the infringer from using the mark
C. An additional amount of damages computed as a multiplier of 5 times the original damages
D. Damages, an injunction prohibiting the infringer from using the mark, and additional
damages based on a multiplier of 5 times the original damages
E. Damages and an injunction prohibiting the infringer from using the mark

If a mark is registered, the holder of the mark may recover damages from an infringer who
uses it to pass off goods as being those of the mark owner, and the owner may also obtain an
injunction prohibiting the infringer from using the mark.

AACSB: Analytical Thinking


Accessibility: Keyboard Navigation
Blooms: Understand
Difficulty: 2 Medium
Learning Objective: 08-07 What are the similarities and differences among the methods for protecting various types of
intellectual property?
Topic: Intellectual Property

52. If a trademark is unregistered, which of the following may the holder recover when an infringer
uses the mark to pass off goods as being those of the mark owner?

A. Damages.
B. An injunction prohibiting the infringer from using the mark.
C. An additional amount of damages computed as a multiplier of 5 times the original
damages.
D. Damages, an injunction prohibiting the infringer from using the mark, and additional
damages based on a multiplier of 5 times the original damages.
E. Damages and an injunction prohibiting the infringer from using the mark.

If a mark is unregistered, only an injunction prohibiting the infringer from using the mark may
be obtained.

AACSB: Analytical Thinking


Accessibility: Keyboard Navigation
Blooms: Understand
Difficulty: 2 Medium
Learning Objective: 08-07 What are the similarities and differences among the methods for protecting various types of
intellectual property?
Topic: Intellectual Property

8-43
Copyright © 2016 McGraw-Hill Education. All rights reserved. No reproduction or distribution without the prior written consent of
McGraw-Hill Education.
53. Once a trademark is registered, when must it initially be renewed?

A. Between the first and second years


B. Between the second and third years
C. Between the third and fourth years
D. Between the fourth and fifth years
E. Between the fifth and sixth years

Once the mark is registered, the registration must be renewed between the fifth and sixth
years.

AACSB: Analytical Thinking


Accessibility: Keyboard Navigation
Blooms: Remember
Difficulty: 1 Easy
Learning Objective: 08-07 What are the similarities and differences among the methods for protecting various types of
intellectual property?
Topic: Intellectual Property

54. After the initial renewal, assuming a trademark was initially registered after 1990, how often
must the trademark be renewed?

A. Every nine years


B. Every ten years
C. Every eleven years
D. Every twelve years
E. Every thirteen years

After that initial renewal, the mark holder must renew the registration every 10 years. (If the
mark was initially registered before 1990, however, renewal is necessary only every 20 years.)

AACSB: Analytical Thinking


Accessibility: Keyboard Navigation
Blooms: Remember
Difficulty: 1 Easy
Learning Objective: 08-07 What are the similarities and differences among the methods for protecting various types of
intellectual property?
Topic: Intellectual Property

8-44
Copyright © 2016 McGraw-Hill Education. All rights reserved. No reproduction or distribution without the prior written consent of
McGraw-Hill Education.
55. In which of the following orders of ascending strength may trademarks fall?

A. Generic, suggestive, descriptive, arbitrary, or fanciful


B. Descriptive, generic, suggestive, arbitrary, or fanciful
C. Arbitrary or fanciful, generic, descriptive, suggestive
D. Generic, descriptive, suggestive, arbitrary, or fanciful
E. Suggestive, generic, descriptive, arbitrary, or fanciful

A mark can fall into one of four general categories, which, in order of ascending strength, are
(1) generic (is entitled to no protection); (2) descriptive (has a secondary meaning and informs
public about uses of product); (3) suggestive (suggests qualities of product but requires
imagination); and (4) arbitrary or fanciful (does not describe the qualities of the product).

AACSB: Analytical Thinking


Accessibility: Keyboard Navigation
Blooms: Understand
Difficulty: 2 Medium
Learning Objective: 08-07 What are the similarities and differences among the methods for protecting various types of
intellectual property?
Topic: Intellectual Property

56. Which of the following is the way people and businesses are located on the web?

A. Through product names


B. Through Internet names
C. Through domain names
D. Through trademark names
E. Through search engines

Domain names are important because they are the addresses by which people and
businesses are located on the web.

AACSB: Technology
Accessibility: Keyboard Navigation
Blooms: Understand
Difficulty: 2 Medium
Learning Objective: 08-07 What are the similarities and differences among the methods for protecting various types of
intellectual property?
Topic: Intellectual Property

8-45
Copyright © 2016 McGraw-Hill Education. All rights reserved. No reproduction or distribution without the prior written consent of
McGraw-Hill Education.
57. With which of the following does a government website address end?

A. .gov
B. .edu
C. .net
D. .org
E. .com

If it is a government site, the domain name will end in gov.

AACSB: Technology
Accessibility: Keyboard Navigation
Blooms: Remember
Difficulty: 1 Easy
Learning Objective: 08-07 What are the similarities and differences among the methods for protecting various types of
intellectual property?
Topic: Intellectual Property

58. Which of the following is responsible for registering domain names for Internet use?

A. Internet Solutions Inc.


B. Domain Names Co.
C. Internet Domain Inc.
D. Network Solutions Inc.
E. Findability Inc.

Network Solutions Inc. (NSI), which is funded by the National Science Foundation, is
responsible for registering domain names.

AACSB: Technology
Accessibility: Keyboard Navigation
Blooms: Remember
Difficulty: 1 Easy
Learning Objective: 08-07 What are the similarities and differences among the methods for protecting various types of
intellectual property?
Topic: Intellectual Property

8-46
Copyright © 2016 McGraw-Hill Education. All rights reserved. No reproduction or distribution without the prior written consent of
McGraw-Hill Education.
59. A registrant may lose registration of a domain name for Internet usage by not using it for more
than ___________ days.

A. 30
B. 60
C. 90
D. 120
E. 180

A party may lose registration of a name by not using it for more than 90 days.

AACSB: Technology
Accessibility: Keyboard Navigation
Blooms: Remember
Difficulty: 1 Easy
Learning Objective: 08-07 What are the similarities and differences among the methods for protecting various types of
intellectual property?
Topic: Intellectual Property

60. Which of the following provides that a portion of a copyrighted work may be reproduced for
purposes of criticism, comment, news reporting, teaching, scholarships, and research?

A. The limited-use doctrine


B. The copyright-use doctrine
C. The fair-use doctrine
D. The trade-use doctrine
E. None of these because there is no such provision

The fair-use doctrine provides that a portion of a copyrighted work may be reproduced for
purposes of "criticism, comment, news reporting, teaching (including multiple copies for
classroom use), scholarships, and research."

AACSB: Analytical Thinking


Accessibility: Keyboard Navigation
Blooms: Understand
Difficulty: 2 Medium
Learning Objective: 08-07 What are the similarities and differences among the methods for protecting various types of
intellectual property?
Topic: Intellectual Property

8-47
Copyright © 2016 McGraw-Hill Education. All rights reserved. No reproduction or distribution without the prior written consent of
McGraw-Hill Education.
61. A patent protects a product, process, invention, machine, or a(n) ______.

A. trademark
B. copyright
C. plant produced by asexual reproduction
D. book that has not yet been submitted for copyright protection
E. idea

A patent protects a product, process, invention, machine, or a plant produced by asexual


reproduction.

AACSB: Analytical Thinking


Accessibility: Keyboard Navigation
Blooms: Understand
Difficulty: 2 Medium
Learning Objective: 08-07 What are the similarities and differences among the methods for protecting various types of
intellectual property?
Topic: Intellectual Property

62. Which of the following criteria must be satisfied in order for a patent, other than a design
patent, to be granted?

A. The object of the patent must be novel.


B. The object of the patent must be useful.
C. The object of the patent must be nonobvious.
D. The object of the patent must be novel but it does not have to be nonobvious.
E. The object of the patent must be novel, useful, and nonobvious.

For a patent to be granted, the following three criteria must be satisfied: the object of the
patent must be novel, or new; the object must be useful, unless it is a design; and the object
must be nonobvious.

AACSB: Analytical Thinking


Accessibility: Keyboard Navigation
Blooms: Understand
Difficulty: 2 Medium
Learning Objective: 08-07 What are the similarities and differences among the methods for protecting various types of
intellectual property?
Topic: Intellectual Property

8-48
Copyright © 2016 McGraw-Hill Education. All rights reserved. No reproduction or distribution without the prior written consent of
McGraw-Hill Education.
63. When a patent is issued for an object, it gives its holder the exclusive right to produce, sell,
and use the object of the patent for __________ years from the date of application.

A. ten
B. twenty
C. thirty
D. forty
E. fifty

When a patent is issued for an object, it gives its holder the exclusive right to produce, sell,
and use the object of the patent for 20 years from the date of application.

AACSB: Analytical Thinking


Accessibility: Keyboard Navigation
Blooms: Remember
Difficulty: 1 Easy
Learning Objective: 08-07 What are the similarities and differences among the methods for protecting various types of
intellectual property?
Topic: Intellectual Property

64. Which of the following is true regarding trade-secret protection?

A. A trade secret is protected from unlawful appropriation by competitors as long as it is kept


secret and consists of elements not generally known in the trade.
B. A trade secret is protected from unlawful appropriation by competitors for ten years so long
as no competitor gains access to the secret through lawful means.
C. A trade secret is protected from unlawful appropriation by competitors for twenty years so
long as no competitor gains access to the secret through lawful or unlawful means.
D. A trade secret is protected from unlawful appropriation by competitors for thirty years.
E. A trade secret is protected from unlawful appropriation by competitors for seventy years.

A trade secret is protected by the common law from unlawful appropriation by competitors as
long as it is kept secret and consists of elements not generally known in the trade.

AACSB: Analytical Thinking


Accessibility: Keyboard Navigation
Blooms: Understand
Difficulty: 2 Medium
Learning Objective: 08-07 What are the similarities and differences among the methods for protecting various types of
intellectual property?
Topic: Intellectual Property

8-49
Copyright © 2016 McGraw-Hill Education. All rights reserved. No reproduction or distribution without the prior written consent of
McGraw-Hill Education.
65. Which of the following is false regarding trade-secret protection?

A. Competitors may not legally discover trade secrets by doing reverse engineering.
B. Competitors may discover secrets by going on public tours of plants and observing the use
of the trade secret.
C. Lawful discovery of a trade secret means there is no longer a trade secret to be protected.
D. An invention may be considered a trade secret.
E. A design may be considered a trade secret.

Competitors may discover a "secret" by any lawful means, such as by doing reverse
engineering or by going on public tours of plants and observing the use of trade secrets.

AACSB: Analytical Thinking


Accessibility: Keyboard Navigation
Blooms: Understand
Difficulty: 2 Medium
Learning Objective: 08-07 What are the similarities and differences among the methods for protecting various types of
intellectual property?
Topic: Intellectual Property

66. To stop a competitor form using a trade secret, the plaintiff must prove that a trade secret
actually existed, that the defendant acquired the trade secret unlawfully and ___________.

A. it is the plaintiff's sole trade secret


B. the inventor successfully patented the trade secret
C. the defendant published the trade secret
D. the defendant used the trade secret without the plaintiff's permission
E. the plaintiff had the trade secret for less than 20 years

To stop a competitor from using a trade secret, the plaintiff must prove that a trade secret
actually existed, that the defendant acquired the trade secret unlawfully and the defendant
used the trade secret without the plaintiff's permission.

AACSB: Analytical Thinking


Accessibility: Keyboard Navigation
Blooms: Understand
Difficulty: 2 Medium
Learning Objective: 08-07 What are the similarities and differences among the methods for protecting various types of
intellectual property?
Topic: Intellectual Property

8-50
Copyright © 2016 McGraw-Hill Education. All rights reserved. No reproduction or distribution without the prior written consent of
McGraw-Hill Education.
67. In the intellectual property arena, the term ______ refers to the overall appearance and image
of a product.

A. product appearance
B. trade dress
C. design fabrication
D. trade appearance
E. product application

The term trade dress refers to the overall appearance and image of a product.

AACSB: Analytical Thinking


Accessibility: Keyboard Navigation
Blooms: Remember
Difficulty: 1 Easy
Learning Objective: 08-07 What are the similarities and differences among the methods for protecting various types of
intellectual property?
Topic: Intellectual Property

68. Which of the following is a type of property that can be touched?

A. Tangible
B. Intangible
C. Substantive
D. Productive
E. All of these

Tangible property is property that you can see or touch.

AACSB: Analytical Thinking


Accessibility: Keyboard Navigation
Blooms: Remember
Difficulty: 1 Easy
Learning Objective: 08-04 What are the types of personal property?
Topic: Personal Property

8-51
Copyright © 2016 McGraw-Hill Education. All rights reserved. No reproduction or distribution without the prior written consent of
McGraw-Hill Education.
69. Bank accounts, stocks, and insurance policies are examples of which of the following types of
property?

A. Tangible
B. Intangible
C. Substantive
D. Productive
E. All of these

Intangible property includes such items as bank accounts, stocks, and insurance policies.

AACSB: Analytical Thinking


Accessibility: Keyboard Navigation
Blooms: Understand
Difficulty: 2 Medium
Learning Objective: 08-04 What are the types of personal property?
Topic: Personal Property

70. Which of the following is a way in which a gift differs from a purchase as a way of transferring
ownership?

A. Once delivered a gift may be taken back by the person who gave the gift, whereas that is
not true when an item is purchased.
B. No consideration is needed for a gift.
C. No written contract is needed for a gift, whereas it is needed for a purchase.
D. No certificate of title is needed for a gift, whereas it is needed for a purchase.
E. All of these.

When transfer of the property is by purchase, the acquiring party gives some consideration to
the seller in exchange for title to the property; but that is not true when gifts are involved.

AACSB: Analytical Thinking


Accessibility: Keyboard Navigation
Blooms: Understand
Difficulty: 2 Medium
Learning Objective: 08-05 How do you transfer personal property?
Topic: Personal Property

8-52
Copyright © 2016 McGraw-Hill Education. All rights reserved. No reproduction or distribution without the prior written consent of
McGraw-Hill Education.
71. Which of the following is needed for a valid gift?

A. Delivery
B. Donative intent
C. Acceptance
D. Delivery, donative intent, and acceptance
E. Delivery and donative intent, but not acceptance

For a valid gift, there must be delivery, donative intent, and acceptance.

AACSB: Analytical Thinking


Accessibility: Keyboard Navigation
Blooms: Understand
Difficulty: 2 Medium
Learning Objective: 08-05 How do you transfer personal property?
Topic: Personal Property

72. Property that the original owner has discarded is ______ property.

A. mislaid
B. tossed
C. discarded
D. abandoned
E. terminated

Property that the original owner has discarded is abandoned property.

AACSB: Analytical Thinking


Accessibility: Keyboard Navigation
Blooms: Remember
Difficulty: 1 Easy
Learning Objective: 08-05 How do you transfer personal property?
Topic: Personal Property

8-53
Copyright © 2016 McGraw-Hill Education. All rights reserved. No reproduction or distribution without the prior written consent of
McGraw-Hill Education.
73. Which of the following is property that the true owner has unknowingly or accidentally dropped
or left somewhere?

A. Lost
B. Mislaid
C. Discarded
D. Abandoned
E. Terminated

Lost property is property that the true owner has unknowingly or accidentally dropped or left
somewhere.

AACSB: Analytical Thinking


Accessibility: Keyboard Navigation
Blooms: Remember
Difficulty: 1 Easy
Learning Objective: 08-05 How do you transfer personal property?
Topic: Personal Property

74. Which of the following is property that the true owner has intentionally placed somewhere but
has forgotten its location?

A. Lost
B. Mislaid
C. Discarded
D. Abandoned
E. Terminated

Mislaid property differs from lost property in that the owner has intentionally placed the
property somewhere but has forgotten its location.

AACSB: Analytical Thinking


Accessibility: Keyboard Navigation
Blooms: Remember
Difficulty: 1 Easy
Learning Objective: 08-05 How do you transfer personal property?
Topic: Personal Property

8-54
Copyright © 2016 McGraw-Hill Education. All rights reserved. No reproduction or distribution without the prior written consent of
McGraw-Hill Education.
75. Which of the following is true regarding lost property?

A. In most states the finder of lost property has title to the lost good against all including the
person who lost the property.
B. In most states the finder of lost property has title to the lost good against all except the
person who lost the property.
C. In most states the finder of lost property has no title to the lost good because the good is to
be turned over to the police department for later sale if the true owner cannot be located.
D. In all states the finder of property has an obligation to put a notice in the local paper looking
for the true owner before the finder of the property can claim ownership.
E. In most states the finder of lost property has title to the lost good against all except the
person who lost the property, but that right only arises after the finder of the property can
establish that the finder spent at least one-third of the fair market value of the property in
locating the owner.

In most states, the finder of lost property has title to the lost good against all except the true
owner.

AACSB: Analytical Thinking


Accessibility: Keyboard Navigation
Blooms: Remember
Difficulty: 1 Easy
Learning Objective: 08-05 How do you transfer personal property?
Topic: Personal Property

76. Disputed Ring. While working in the yard, Tina found a beat up ring. Becca, an eighteen-year
old teenager and neighbor, came over to visit and liked the ring. Tina said, "You can have this
old thing if you would like." Becca replied, "I really like it - Maybe it's a real diamond!" Tina
laughingly told her that there was a one in a billion chance of that and that Tina was more
likely to win the lottery. A few months later Becca ran over to Tina's house and told Tina that
the ring was actually a diamond worth thousands! Becca gave Tina the ring to examine. Tina
put it in her pocket and told Becca that she would never have given it to her if she had realized
its value and that possession was back where it had always belonged. Tina also told Becca
that Becca failed legally to accept the gift because she did not know its true value and also
because Tina did not sign any document turning over title. Becca sues. Which of the following
describes the type of gift at issue?

A. A gift causa mortis


B. A gift inter vivos
C. A gift inter mortis
D. A gift causa vivos
E. A donative gift

Inter vivos gifts are gifts made by a person during his or her lifetime.

AACSB: Analytical Thinking


Accessibility: Keyboard Navigation
Blooms: Apply
Difficulty: 1 Easy
Learning Objective: 08-05 How do you transfer personal property?
Topic: Personal Property

8-55
Copyright © 2016 McGraw-Hill Education. All rights reserved. No reproduction or distribution without the prior written consent of
McGraw-Hill Education.
77. Disputed Ring. While working in the yard, Tina found a beat up ring. Becca, an eighteen-year
old teenager and neighbor, came over to visit and liked the ring. Tina said, "You can have this
old thing if you would like." Becca replied, "I really like it - Maybe it's a real diamond!" Tina
laughingly told her that there was a one in a billion chance of that and that Tina was more
likely to win the lottery. A few months later Becca ran over to Tina's house and told Tina that
the ring was actually a diamond worth thousands! Becca gave Tina the ring to examine. Tina
put it in her pocket and told Becca that she would never have given it to her if she had realized
its value and that possession was back where it had always belonged. Tina also told Becca
that Becca failed legally to accept the gift because she did not know its true value and also
because Tina did not sign any document turning over title. Becca sues. Which of the following
is true regarding Tina's statement that she was entitled to the ring because she would not
have given it to Becca if she had known the true value?

A. Tina is entitled to ownership of the ring if she can prove that she did not realize its true
value.
B. Tina is entitled to ownership of the ring only if she can prove that she did not realize its true
value, and also that there was a difference of at least $1,000 between what she believed
the value to be and its true value.
C. Tina is entitled to ownership of the ring only if she can prove that she did not realize its true
value and also that there was a difference of at least $10,000 between what she believed
the value to be and its true value.
D. Tina is entitled to ownership of the ring but only because neither she nor Becca realized its
true value.
E. Being ignorant of the value of the ring does not entitle Tina to ownership of it.

The elements necessary for a valid gift are delivery, donative intent to make an immediate gift,
and acceptance.

AACSB: Reflective Thinking


Accessibility: Keyboard Navigation
Blooms: Evaluate
Difficulty: 3 Hard
Learning Objective: 08-05 How do you transfer personal property?
Topic: Personal Property

8-56
Copyright © 2016 McGraw-Hill Education. All rights reserved. No reproduction or distribution without the prior written consent of
McGraw-Hill Education.
78. Disputed Ring. While working in the yard, Tina found a beat up ring. Becca, an eighteen-year
old teenager and neighbor, came over to visit and liked the ring. Tina said, "You can have this
old thing if you would like." Becca replied, "I really like it - Maybe it's a real diamond!" Tina
laughingly told her that there was a one in a billion chance of that and that Tina was more
likely to win the lottery. A few months later Becca ran over to Tina's house and told Tina that
the ring was actually a diamond worth thousands! Becca gave Tina the ring to examine. Tina
put it in her pocket and told Becca that she would never have given it to her if she had realized
its value and that possession was back where it had always belonged. Tina also told Becca
that Becca failed legally to accept the gift because she did not know its true value and also
because Tina did not sign any document turning over title. Becca sues. Which of the following
is true regarding Tina's statement that Becca could not have validly accepted the ring?

A. If the ring is valued at $500 or over, then Tina is correct that Becca did not validly accept
the ring because a writing would have been needed to validate acceptance.
B. If the ring is valued at $1,000 or over, then Tina is correct that Becca did not validly accept
the ring because a writing would have been needed to validate acceptance.
C. If the ring is valued at $10,000 or over, then Tina is correct that Becca did not validly
accept the ring because a writing would have been needed to validate acceptance.
D. Tina is correct that Becca could not have validly accepted the ring when she did not
understand its true value.
E. Tina is incorrect, and Becca validly accepted the ring as a gift.

A transfer of ownership of the ring required no formalities and acceptance is satisfied by the
willingness of the donee to take the gift from the donor.

AACSB: Reflective Thinking


Accessibility: Keyboard Navigation
Blooms: Apply
Difficulty: 3 Hard
Learning Objective: 08-05 How do you transfer personal property?
Topic: Personal Property

8-57
Copyright © 2016 McGraw-Hill Education. All rights reserved. No reproduction or distribution without the prior written consent of
McGraw-Hill Education.
79. Easement Dispute. Sally buys a house from Bob that borders on a lake. Her deed gives her
the most complete estate a person may have. She is in her bathing suit sunning one day when
her neighbor, Fred cuts through her yard. Sally tells him not to do that anymore. Fred informs
her that he had previously purchased and properly recorded a right to cut across her yard to
get lake access. Sally asks you what she should do to try to get rid of Fred because she
doesn't like him cutting through her yard when she is sunning. What type of estate did Sally
purchase from Bob?

A. A life absolute estate


B. A conditional estate
C. A fee simple absolute estate
D. A limited estate
E. A subject estate

A fee simple absolute is the most complete estate a person may have.

AACSB: Reflective Thinking


Accessibility: Keyboard Navigation
Blooms: Apply
Difficulty: 3 Hard
Learning Objective: 08-03 What are the interests in real property that one can hold?
Topic: Real Property

80. Easement Dispute. Sally buys a house from Bob that borders on a lake. Her deed gives her
the most complete estate a person may have. She is in her bathing suit sunning one day when
her neighbor, Fred cuts through her yard. Sally tells him not to do that anymore. Fred informs
her that he had previously purchased and properly recorded a right to cut across her yard to
get lake access. Sally asks you what she should do to try to get rid of Fred because she
doesn't like him cutting through her yard when she is sunning. What is the type of right that
Fred is exercising by cutting through the yard called?

A. A profit
B. A license
C. A condition
D. An easement
E. A breach of warranty

An easement is an irrevocable right to use some part of another's land for a specific purpose
without taking anything from it.

AACSB: Reflective Thinking


Accessibility: Keyboard Navigation
Blooms: Apply
Difficulty: 3 Hard
Learning Objective: 08-03 What are the interests in real property that one can hold?
Topic: Real Property

8-58
Copyright © 2016 McGraw-Hill Education. All rights reserved. No reproduction or distribution without the prior written consent of
McGraw-Hill Education.
81. Easement Dispute. Sally buys a house from Bob that borders on a lake. Her deed gives her
the most complete estate a person may have. She is in her bathing suit sunning one day when
her neighbor, Fred cuts through her yard. Sally tells him not to do that anymore. Fred informs
her that he had previously purchased and properly recorded a right to cut across her yard to
get lake access. Sally asks you what she should do to try to get rid of Fred because she
doesn't like him cutting through her yard when she is sunning. What is the best advice you can
give Sally regarding getting rid of Fred?

A. To call the police when he shows up because any rights he had, terminated when Sally
bought the property.
B. To sell her property and move on because she has absolutely no hope.
C. To put up with Fred for one year because his interest could only last that long after her
purchase.
D. To put up with Fred for three years because his interest could only last that long after her
purchase.
E. To reach some sort of agreement with him, possibly by paying him to give up the
easement.

Easements can be terminated, with the most common method of termination being by
agreement.

AACSB: Reflective Thinking


Accessibility: Keyboard Navigation
Blooms: Evaluate
Difficulty: 3 Hard
Learning Objective: 08-03 What are the interests in real property that one can hold?
Topic: Real Property

8-59
Copyright © 2016 McGraw-Hill Education. All rights reserved. No reproduction or distribution without the prior written consent of
McGraw-Hill Education.
82. Wedding Photos. Bobby took a number of wedding photos at Jill's wedding. He was paid as
the photographer. On all of the photographs, he appropriately noted in the bottom right-hand
corner the necessary information showing that he was claiming copyright protection. Jill came
to see Bobby three years after the initial photographs were taken and requested that he grant
her permission to run off as many copies as she wanted at the local photo shop from the
pictures that she initially purchased. The photo shop had refused to reproduce the
photographs without his permission. When he refused to give her permission to do so, Jill
started a heated argument. She told Bobby that photographs are not entitled to copyright
protection. She also told him that even if he was correct that there was some copyright
protection, she was engaged in fair use; and that, in any event, damages for copyright
infringement are unavailable. Which of the following is correct regarding Jill's claim that
photographs are not subject to copyright protection?

A. She is correct. Photographs are not subject to copyright protection even if taken by a
professional photographer.
B. She is correct but only because family pictures are involved. Family pictures may not be
the subject of copyright, but landscape photographs may be the subject of copyright
protection.
C. She is partially correct. Bobby was entitled to copyright protection on the first picture. After
Jill purchased the first picture, however, she could make as many copies as she wanted.
D. She is correct only because Bobby had not registered the photographs for copyright
protection.
E. She is incorrect.

There are three criteria for a work to be copyrightable. First, it must be fixed, which means set
out in a tangible medium of expression. Second, it must be original. Third, it must be creative.

AACSB: Reflective Thinking


Accessibility: Keyboard Navigation
Blooms: Apply
Difficulty: 3 Hard
Learning Objective: 08-07 What are the similarities and differences among the methods for protecting various types of
intellectual property?
Topic: Intellectual Property

8-60
Copyright © 2016 McGraw-Hill Education. All rights reserved. No reproduction or distribution without the prior written consent of
McGraw-Hill Education.
83. Wedding Photos. Bobby took a number of wedding photos at Jill's wedding. He was paid as
the photographer. On all of the photographs, he appropriately noted in the bottom right-hand
corner the necessary information showing that he was claiming copyright protection. Jill came
to see Bobby three years after the initial photographs were taken and requested that he grant
her permission to run off as many copies as she wanted at the local photo shop from the
pictures that she initially purchased. The photo shop had refused to reproduce the
photographs without his permission. When he refused to give her permission to do so, Jill
started a heated argument. She told Bobby that photographs are not entitled to copyright
protection. She also told him that even if he was correct that there was some copyright
protection, she was engaged in fair use; and that, in any event, damages for copyright
infringement are unavailable. Which of the following is true regarding Jill's claim that she was
engaged in fair use?

A. It is likely that she would win on the claim because she does not seek to use the
photographs in a business pursuit.
B. It is likely that she would win on the claim because she only seeks to copy what she has
already purchased.
C. This issue would not come up because Bobby is not entitled to copyright protection on the
photographs.
D. She would lose because the fair-use doctrine is only available to educators.
E. She would likely lose under the weighing test mandated by the Copyright Act.

In determining whether the fair-use doctrine provides a valid defense to a claim of copyright
infringement, Section 107 of the Copyright Act requires that the court weigh the following four
factors: the purpose and character of the use, the nature of the copyrighted work, the amount
and substantiality of the portion used in relation to the copyrighted work as a whole, and the
effect of the use on the potential market for or value of the copyrighted work.

AACSB: Reflective Thinking


Accessibility: Keyboard Navigation
Blooms: Apply
Difficulty: 3 Hard
Learning Objective: 08-07 What are the similarities and differences among the methods for protecting various types of
intellectual property?
Topic: Intellectual Property

8-61
Copyright © 2016 McGraw-Hill Education. All rights reserved. No reproduction or distribution without the prior written consent of
McGraw-Hill Education.
84. Wedding Photos. Bobby took a number of wedding photos at Jill's wedding. He was paid as
the photographer. On all of the photographs, he appropriately noted in the bottom right-hand
corner the necessary information showing that he was claiming copyright protection. Jill came
to see Bobby three years after the initial photographs were taken and requested that he grant
her permission to run off as many copies as she wanted at the local photo shop from the
pictures that she initially purchased. The photo shop had refused to reproduce the
photographs without his permission. When he refused to give her permission to do so, Jill
started a heated argument. She told Bobby that photographs are not entitled to copyright
protection. She also told him that even if he was correct that there was some copyright
protection, she was engaged in fair use; and that, in any event, damages for copyright
infringement are unavailable. Which of the following is correct regarding Jill's claim that
damages for copyright infringement are unavailable?

A. Damages are available, but a copyrighted work must be registered in order for the creator
to recover damages from infringement.
B. It is not necessary that a copyrighted work be registered in order for the creator to obtain
damages.
C. Damages for copyright infringement are only available if two businesses are involved, not
in disputes involving an individual such as Jill.
D. Some material that is subject to copyright must be registered before its creator may
recover damages for infringement, but that is not true for photographs.
E. She is correct that damages are unavailable in copyright infringement actions because only
injunctive relief is available.

For the creator to be able to not only seek an injunction but also recover damages arising from
the infringement, the copyrighted work must be registered.

AACSB: Reflective Thinking


Accessibility: Keyboard Navigation
Blooms: Apply
Difficulty: 3 Hard
Learning Objective: 08-07 What are the similarities and differences among the methods for protecting various types of
intellectual property?
Topic: Intellectual Property

8-62
Copyright © 2016 McGraw-Hill Education. All rights reserved. No reproduction or distribution without the prior written consent of
McGraw-Hill Education.
85. Wedding Photos. Bobby took a number of wedding photos at Jill's wedding. He was paid as
the photographer. On all of the photographs, he appropriately noted in the bottom right-hand
corner the necessary information showing that he was claiming copyright protection. Jill came
to see Bobby three years after the initial photographs were taken and requested that he grant
her permission to run off as many copies as she wanted at the local photo shop from the
pictures that she initially purchased. The photo shop had refused to reproduce the
photographs without his permission. When he refused to give her permission to do so, Jill
started a heated argument. She told Bobby that photographs are not entitled to copyright
protection. She also told him that even if he was correct that there was some copyright
protection, she was engaged in fair use; and that, in any event, damages for copyright
infringement are unavailable. If Bobby decides to register the photographs under copyright
law, how would he go about doing so?

A. He would register by filing a form with the Register of Copyright and providing two copies
of the copyrighted materials to the Library of Congress.
B. He would register by filing a form with Register of Copyright only.
C. He would register by providing two copies of the copyrighted materials to the Library of
Congress only.
D. He would register by affixing the appropriate symbol at the bottom of the photograph
followed by the first date of publication and his name.
E. There is nothing he can do because the photographs are not subject to copyright
protection.

One may register a work by filing a form with the Register of Copyright and providing two
copies of the copyrighted material to the Library of Congress.

AACSB: Analytical Thinking


Accessibility: Keyboard Navigation
Blooms: Evaluate
Difficulty: 3 Hard
Learning Objective: 08-07 What are the similarities and differences among the methods for protecting various types of
intellectual property?
Topic: Intellectual Property

8-63
Copyright © 2016 McGraw-Hill Education. All rights reserved. No reproduction or distribution without the prior written consent of
McGraw-Hill Education.
86. Scuba Diving. Marcy invented a new type of mask for scuba divers that was not subject to
fogging. She agrees to allow Jenny to manufacture and sell the mask. She receives a sum of
money for every mask that Jenny sells. Marcy also entered into an agreement with Frank to
allow him to sell the masks but only if he also purchased non-patented diving suits from
Marcy. All parties proceeded to do very well with their sales.
Marcy's agreement with Jenny allowing Jenny to sell the mask is referred to as which of the
following?

A. A license
B. A patent agreement
C. A trade agreement
D. An illegal agreement
E. A franchise agreement

The holder of the patent may license, or allow others to manufacture and sell, the patented
object.

AACSB: Reflective Thinking


Accessibility: Keyboard Navigation
Blooms: Apply
Difficulty: 3 Hard
Learning Objective: 08-07 What are the similarities and differences among the methods for protecting various types of
intellectual property?
Topic: Intellectual Property

87. Scuba Diving. Marcy invented a new type of mask for scuba divers that was not subject to
fogging. She agrees to allow Jenny to manufacture and sell the mask. She receives a sum of
money for every mask that Jenny sells. Marcy also entered into an agreement with Frank to
allow him to sell the masks but only if he also purchased non-patented diving suits from
Marcy. All parties proceeded to do very well with their sales.
Payments that Marcy would receive from Jenny for the sale of the mask are referred to as
which of the following?

A. Profits
B. Receipts
C. Royalties
D. Payoffs
E. Illegal

In most cases, patents are licensed in exchange for the payment of royalties, a sum of money
paid for each use of the patented process.

AACSB: Reflective Thinking


Accessibility: Keyboard Navigation
Blooms: Apply
Difficulty: 3 Hard
Learning Objective: 08-07 What are the similarities and differences among the methods for protecting various types of
intellectual property?
Topic: Intellectual Property

8-64
Copyright © 2016 McGraw-Hill Education. All rights reserved. No reproduction or distribution without the prior written consent of
McGraw-Hill Education.
88. Scuba Diving. Marcy invented a new type of mask for scuba divers that was not subject to
fogging. She agrees to allow Jenny to manufacture and sell the mask. She receives a sum of
money for every mask that Jenny sells. Marcy also entered into an agreement with Frank to
allow him to sell the masks but only if he also purchased non-patented diving suits from
Marcy. All parties proceeded to do very well with their sales.
Which of the following describes the agreement between Marcy and Frank?

A. It is a legal tying arrangement.


B. It is a legal cross-licensing agreement.
C. It is an illegal tying arrangement.
D. It is an illegal cross-licensing agreement.
E. It is both a legal tying and a legal cross-licensing agreement.

A tying arrangement is an illegal agreement in which the sale of one product is tied to the sale
of another.

AACSB: Reflective Thinking


Accessibility: Keyboard Navigation
Blooms: Apply
Difficulty: 3 Hard
Learning Objective: 08-07 What are the similarities and differences among the methods for protecting various types of
intellectual property?
Topic: Intellectual Property

89. Lottery Winnings. Frank, a hypochondriac who was also very compulsive, was having minor
surgery to repair a bone spur on his foot. He had just purchased a lottery ticket for a chance at
the grand prize of $30,000,000. Frank's girlfriend, Bubbles, went with him to the hospital.
While in the waiting room, Frank said to her, "Bubbles, I may not make it out of this bone spur
surgery. Take my lottery ticket. If I don't make it, I hope you win and live it up, but please don't
get another boyfriend." Bubbles replied, "I could never be happy without you." A nurse saw
and heard the whole exchange. Frank came out of the surgery just fine but with a sore foot.
While he was recuperating, that evening, Bubbles watched the lottery drawing and discovered
that Frank's ticket was indeed the winning ticket. She immediately moved out and collected the
winnings. Frank saw her on television with her new boyfriend, George. She appeared to be
very happy. He checked the numbers and discovered that she won off of his ticket. Frank says
that the lottery money is his. Which of the following describes the type of gift at issue?

A. A gift causa mortis


B. A gift inter vivos
C. A gift inter mortis
D. A gift causa vivos
E. A donative gift

A type of gift that can be made is a gift causa mortis, a gift that is made in contemplation of
one's immediate death.

AACSB: Reflective Thinking


Accessibility: Keyboard Navigation
Blooms: Evaluate
Difficulty: 3 Hard
Learning Objective: 08-05 How do you transfer personal property?
Topic: Personal Property

8-65
Copyright © 2016 McGraw-Hill Education. All rights reserved. No reproduction or distribution without the prior written consent of
McGraw-Hill Education.
90. Lottery Winnings. Frank, a hypochondriac who was also very compulsive, was having minor
surgery to repair a bone spur on his foot. He had just purchased a lottery ticket for a chance at
the grand prize of $30,000,000. Frank's girlfriend, Bubbles, went with him to the hospital.
While in the waiting room, Frank said to her, "Bubbles, I may not make it out of this bone spur
surgery. Take my lottery ticket. If I don't make it, I hope you win and live it up, but please don't
get another boyfriend." Bubbles replied, "I could never be happy without you." A nurse saw
and heard the whole exchange. Frank came out of the surgery just fine but with a sore foot.
While he was recuperating, that evening, Bubbles watched the lottery drawing and discovered
that Frank's ticket was indeed the winning ticket. She immediately moved out and collected the
winnings. Frank saw her on television with her new boyfriend, George. She appeared to be
very happy. He checked the numbers and discovered that she won off of his ticket. Frank says
that the lottery money is his. Which of the following is true regarding Frank's statement at the
hospital regarding Bubbles taking the ticket?

A. At that point he made a valid gift that could not be revoked.


B. He did not make a gift because he placed a condition on it.
C. He made an irrevocable gift at that time only if Bubbles never had another boyfriend;
otherwise, she had to give the ticket and any resulting cash to Frank.
D. He made an irrevocable gift at that time if Bubbles refrained from having another boyfriend
until she cashed the ticket; and, after the ticket was converted, the condition no longer
applied.
E. None of these.

A gift causa mortis can be revoked.

AACSB: Reflective Thinking


Accessibility: Keyboard Navigation
Blooms: Evaluate
Difficulty: 3 Hard
Learning Objective: 08-05 How do you transfer personal property?
Topic: Personal Property

8-66
Copyright © 2016 McGraw-Hill Education. All rights reserved. No reproduction or distribution without the prior written consent of
McGraw-Hill Education.
91. Lottery Winnings. Frank, a hypochondriac who was also very compulsive, was having minor
surgery to repair a bone spur on his foot. He had just purchased a lottery ticket for a chance at
the grand prize of $30,000,000. Frank's girlfriend, Bubbles, went with him to the hospital.
While in the waiting room, Frank said to her, "Bubbles, I may not make it out of this bone spur
surgery. Take my lottery ticket. If I don't make it, I hope you win and live it up, but please don't
get another boyfriend." Bubbles replied, "I could never be happy without you." A nurse saw
and heard the whole exchange. Frank came out of the surgery just fine but with a sore foot.
While he was recuperating, that evening, Bubbles watched the lottery drawing and discovered
that Frank's ticket was indeed the winning ticket. She immediately moved out and collected the
winnings. Frank saw her on television with her new boyfriend, George. She appeared to be
very happy. He checked the numbers and discovered that she won off of his ticket. Frank says
that the lottery money is his. Which of the following is true regarding rightful ownership of the
lottery money?

A. Frank's gift was automatically revoked when he recovered, and the lottery ticket and any
proceeds are validly his.
B. Although the gift was not automatically revoked on his recovery, Frank has the right to
revoke the gift and recover the lottery proceeds.
C. The gift was not automatically revoked on his recovery and since Frank did not revoke the
gift prior to Bubbles cashing the ticket, he has no rights to any funds.
D. Frank has the right to all funds only because Bubbles has a new boyfriend.
E. Frank has no rights because once he gave the ticket to Bubbles, he gave up all his rights
to it.

A gift causa mortis can be revoked any time before the death of the donor and it is
automatically revoked if the donor recovers.

AACSB: Reflective Thinking


Accessibility: Keyboard Navigation
Blooms: Evaluate
Difficulty: 3 Hard
Learning Objective: 08-05 How do you transfer personal property?
Topic: Personal Property

Essay Questions

92. Discuss the extent of ownership rights when a person owns a fee simple absolute in land,
meaning what interests, if any, does the owner have in addition to rights to use the surface of
the property.

Ownership also extends to airspace, water rights, and mineral rights.

AACSB: Analytical Thinking


Blooms: Understand
Difficulty: 2 Medium
Learning Objective: 08-02 How does real property law balance private and public rights?
Topic: Real Property

8-67
Copyright © 2016 McGraw-Hill Education. All rights reserved. No reproduction or distribution without the prior written consent of
McGraw-Hill Education.
93. Identify where a trademark is registered, what one should do when requesting registration and
what the governmental office involved would then do. Identify circumstances under which a
trademark will not be accepted for registration.

Trademarks are registered with the U.S. Patent Office. To register a mark with the Patent
Office, one must submit a drawing of the mark and indicate when it was first used in interstate
commerce and how it was used. The Patent Office conducts an investigation to verify those
facts and will register a trademark as long as it is not generic, descriptive, immoral, deceptive,
the name of a person whose permission has not been obtained, or substantially similar to
another's trademark.

AACSB: Analytical Thinking


Blooms: Understand
Difficulty: 2 Medium
Learning Objective: 08-07 What are the similarities and differences among the methods for protecting various types of
intellectual property?
Topic: Intellectual Property

94. Ellen, who runs a successful nail salon, ran across a very interesting pamphlet on customer
relations. The pamphlet was fifty pages long and cost $70.00. Ellen thought the price was
outrageous. Accordingly, she bought one copy and photocopied the pages that she thought
were pertinent and gave them to her employees. She copied forty of the pages out of the
pamphlet for each of twenty employees. A disgruntled employee informed the publisher of the
pamphlet what Ellen had done and she was charged with copyright infringement. Ellen
defended on the basis of the fair-use doctrine. List the four factors a court would consider in
determining whether a violation occurred and discuss whether or not you think Ellen would
prevail and why.

The four factors that the court would weigh are as follows:

1. The purpose and character of the use.


2. The nature of the copyrighted work.
3. The amount and substantiality of the portion used in relation to the copyrighted work as a
whole.
4. The effect of the use on the potential market for or value of the copyrighted work.

Although, Ellen did not copy the entire pamphlet, she copied a substantial amount and did so
because she did not want to pay the full price for the copyrighted material for all employees.
Her actions had a negative effect on the market for the work and she was not involved in a
nonprofit educational purpose. It is likely that she would be found to have infringed.

AACSB: Reflective Thinking


Blooms: Evaluate
Difficulty: 3 Hard
Learning Objective: 08-07 What are the similarities and differences among the methods for protecting various types of
intellectual property?
Topic: Intellectual Property

8-68
Copyright © 2016 McGraw-Hill Education. All rights reserved. No reproduction or distribution without the prior written consent of
McGraw-Hill Education.
95. What three criteria must be satisfied in order to obtain a patent that is not a design patent?

First, the object of the patent must be novel, or new. Second, the object must be useful, unless
it is a design and must provide some utility to society. Third, the object must be nonobvious.

AACSB: Analytical Thinking


Blooms: Understand
Difficulty: 2 Medium
Learning Objective: 08-07 What are the similarities and differences among the methods for protecting various types of
intellectual property?
Topic: Intellectual Property

96. Cindy sold her old used automobile to Bruce for $1,000. Cindy told Bruce to come and take
the car and that the transfer would be complete. Is Cindy correct, and why or why not?

Cindy is incorrect because in order to transfer a vehicle, a certificate of title must be signed by
the seller, taken to the appropriate government agency, and then reissued in the name of the
new owner.

AACSB: Reflective Thinking


Blooms: Evaluate
Difficulty: 3 Hard
Learning Objective: 08-05 How do you transfer personal property?
Topic: Personal Property

97. Bernice enjoys snooping, particularly through her neighbor's trash. She sneaks out and grabs
the neighbor's trash after they have gone to work but before the trash collector comes. She
goes through it to find out confidential information regarding the neighbors, but never tells
anyone else or uses the information for any purpose. Discuss whether you believe it is legal
for Bernice to go through the neighbor's trash and why or why not. Also, discuss whether you
believe it is ethical for Bernice to go through the neighbor's trash, and why or why not.

Property that the original owner has discarded is abandoned property. Anyone finding such
property becomes its owner by possessing it. Student answers may vary on the ethical issue.
Most students will likely believe that it is unethical to go through the trash of a neighbor even if
the information is not used for any improper purpose.

AACSB: Ethics
Blooms: Evaluate
Difficulty: 3 Hard
Learning Objective: 08-05 How do you transfer personal property?
Topic: Personal Property

8-69
Copyright © 2016 McGraw-Hill Education. All rights reserved. No reproduction or distribution without the prior written consent of
McGraw-Hill Education.

You might also like